Sunteți pe pagina 1din 36

w

w
.
n
e
u
t
r
i
n
o
.
r
o
Societatea de tiine Matematice din Romnia
Filiala Cara-Severin



REVISTA DE
MATEMATIC



A ELEVILOR I PROFESORILOR


DIN JUDEUL
CARA-SEVERIN

Nr. 25, An IX-2008


Editura Neutrino
Reia, 2008
2
2008, Editura Neutrino
Titlul: Revista de matematic a elevilor i profesorilor din judeul
Cara-Severin
I.S.S.N. 1584-9481


Colectivul de redacie

Avrmescu Irina Iatan Rodica
Bdescu Ovidiu Lazarov Mihael
Chi Vasile Mitric Mariana
Dragomir Adriana Moatr Lavinia
Dragomir Lucian Neagoe Petrior
Drghici Mariana Pistril Ion Dumitru
Didraga Iacob Stniloiu Nicolae
Gdea Vasilica andru Marius
Golopena Marius uoi Paul


Redacia

Redactor-ef: Dragomir Lucian
Redactor-ef Adjunct: Bdescu Ovidiu
Redactori principali: Dragomir Adriana
Neagoe Petrior
Stniloiu Nicolae

Responsabil de numr:Adriana Dragomir




2008, Editura Neutrino
Toate drepturile rezervate
Mobil: 0724224400
www.neutrino.ro
E-mail: editura@neutrino.ro
w
w
.
n
e
u
t
r
i
n
o
.
r
o
3

CUPRINS





Din cugetrile lui Nicolae Iorga ........ Pag. 4
Chestiuni metodice, note matematice
Puncte importante n triunghi (I)
(Marina i Mircea Constantinescu)...
Generalizarea unei probleme de concurs
(Nicolae Stniloiu) ......................
Metode de rezolvare a problemelor cu unghiuri n
spaiu (Maria Iancu) ..
Diversitate n curriculumul colar curs opional
(Tudor Deaconu)..
Calculul unor sume cu combinri
(Nicolae Stniloiu)..
Partea ntreag,partea fracionar a unui numr
real (Lucian Dragomir) ..
Olimpiada Naional 2008 pentru clasele V-VI
(Miruna i Loreta Ciulu)..


Pag. 5

Pag.10

Pag.12

Pag.15

Pag.19

Pag.21

Pag.30
Graficul olimpiadelor i concursurilor .

Pag.31

Probleme rezolvate ...
Concursul revistei, ediia a IV-a ...................................
Pag.32
Pag.53
Probleme propuse .
Tabel nominal cu membrii Filialei Cara-Severin ai
S.S.M.R cu starea cotizaiei pe anii 2007, 2008........
Pag.54

Pag.62
Rubrica rezolvitorilor Pag.67



4

Din cugetrile lui Nicolae Iorga

Nu spune niciodat nu se poate, ncepe cu s vedem.

Scrie ca s nu pierzi florile gndului tu pe care, altfel, le ia vntul!

Dac poi merge, de ce s te trti ?

Viaa nu nseamn a tri, ci a ti pentru ce trieti.

Sunt succese care te njosesc i eecuri care te nal.

Nu imita ce nu-i trebuie.

Pentru fiecare om e un drum ctre fericire: acela pe care e chemat s
mearg. Cei mai muli nu-l gsesc niciodat, cei cumini ncearc pn la
moarte, iar cei proti se trntesc la pmnt i plng c sunt nenorocii.

Zburnd sus, te faci nevzut, dar vezi.

Omul cu adevrat bun este doar cel care ar fi putut fi ru i n-a fost.

Un om cult nseamn o minte deschis ctre buntate i frumusee.

coala cea mai bun e aceea n care nvei nainte de toate a nva.

Ataci o prere a unui prost i te trezeti cu prostul ntreg n discuie.

Drumul lung ncepe de unde ai obosit.

Cel care st jos s nu rd de cel care cade de pe vrfuri.

Copiii trebuie crescui pentru ei, nu pentru prini.

Cine uit, nu merit.

coala cea bun e aceea n care i colarul nva pe profesor.
w
w
.
n
e
u
t
r
i
n
o
.
r
o
5

Puncte importante n triunghi
1.Ortocentrul

Marina Constantinescu, Mircea Constantinescu

Exist o seam de puncte remarcabile ntr-un triughi care joac un
rol fundamental n geometria triunghiului. n programa colar n
vigoare se studiaz unele dintre cele mai importante precum: centrul de
greutate, ortocentrul, centrele cercurilor circumscris i nscris n
triunghi, ntre acestea existnd relaii remarcabile. n cadrul acestui
articol ne vom ocupa de unul dintre aceste puncte: ortocentrul unui
triunghi.
Vom stabili cteva proprieti i vom prezenta unele aplicaii.
Admitem cunoscut faptul c ntr-un triunghi nlimile sunt concurente.
Definiie. Punctul de intersecie a nlimilor unui triunghi se
numete ortocentrul triunghiului.
Observaie. Dac triunghiul este ascuitunghic atunci ortocentrul
se afl n interiorul su, dac triunghiul este obtuzunghic atunci
ortocentrul este situat n exteriorul triunghiului, iar n cazul triunghiului
dreptunghic ortocentrul coincide cu vrful unghiului drept.
Concurena nlimilor unui triunghi poate fi folosit la
demonstrarea perpendicularitii unor drepte dup cum vom vedea n
urmtoarele aplicaii.
Problema 1. Unghiurile opuse A i C ale patrulaterului ABCD
sunt drepte. Fie AD CB { } E = i { } AB CD F = . S se arate c
. BD EF
Soluie. Se observ c AE i CF sunt nlimi n BEF , deci D
este ortocentrul triunghiului BEF i atunci . BD EF
Problema 2. Se consider tetraedrul ABCD n care AB CD i
. BC AD S se arate c BD AC .
Soluie. Fie H ortocentrul triunghiului BCD i
/
, BB CD
/
. DD BC Cum CD AB rezult c . CD AH Analog BC AH
deci ( ) AH BCD . Deoarece H este ortocentrul triunghiului BCD
rezult c CH BD . Cum AH BD se obine ( ) BD AHC de unde
concluzia este imediat.
Problema 3. Se consider triunghiul dreptunghic ABC i
, , A B C punctele de tangen ale laturilor ( ) ( ) ( ) , , BC CA AB cu cercul
6
su nscris de centru I . Dreptele BI i CI intersecteaz pe A B
respectiv A C n D i E . Fie { } . H B E C D = S se arate c
. A H B C
Soluie. Deoarece BC BA = rezult c ( ) 1 AC A CA C < < .
Cum ( ) . . EA C EB C LU L rezult c ( ) 2 . CA E CB E < Din ( ) 1
i ( ) 2 obinem AC E CB E < < deci patrulaterul AC EB este
inscriptibil. Aadar . B E A C Analog se deduce c C D A B i
deci H este ortocentrul A B C de unde rezult c . A H B C
Prezentm n continuare unele rezultate remarcabile i utile n
legtur cu ortocentrul unui triunghi.
Propoziia 1
a) Fie
1 1 1
, , A B C interseciile nalimilor , , AA BB CC ale
unui triunghi ABC cu cercul su circumscris. Atunci
1 1 1
, , A B C sunt
simetricele ortocentrului H al triunghiului ABC n raport cu , BC CA
respectiv . AB
b) Cu notaiile anterioare ortocentrul H al triunghiului,
mijlocul M al laturii BC i punctul
2
A diametral opus lui A sunt
coliniare, iar dac O este centrul cercului circumscris atunci
2 . AH OM =
Soluie.a). ( ) ( )

( )
( )
1
1 1
1 .
2
m AC
m A BA m A AC = = < Cum
BB C AA C ( ) . . U U rezult c ( )
1
2 . HBA A AC < < Din ( ) 1 i
( ) 2 deducem c
1
HBA A BA < < i cum
1
BA HA rezult c
1
. HA A A = Analog se obin celelalte dou relaii.
b). ( )
2
90 m A CA =

< deci
2
A C CA i cum BH CA
rezult c BH i
2
A C sunt paralele. Analog CH i
2
A B sunt paralele
deci
2
BA CH este paralelogram i deci
2
, , H M A sunt coliniare i
2
. HM MA = Atunci OM este linie mijlocie n
2
A AH deci
2 . AH OM =
Propoziia 2
Se consider triunghiul ABC i , , A B C picioarele nlimilor, H
ortocentrul triunghiului, , , A B C mijloacele laturilor i
1 1 1
, , A B C
w
w
.
n
e
u
t
r
i
n
o
.
r
o
7
mijloacele segmentelor [ ] [ ] [ ] , , AH BH CH . Atunci cele nou puncte
sunt situate pe acelai cerc (cercul lui Euler).
Soluia 1.
1 1
A B i A B fiind linii mijlocii n AHB respectiv
ACB se obine c
1 1
A B A B este paralelogram ( ) 1 .
Cum
1
A B este linie mijlocie n AHC . rezult c
1
A B i CH sunt
paralele deci
1
A B AB deci ( )
1 1 1
2 . A B A B Relaiile ( ) 1 i ( ) 2 arat
c
1 1
A B A B este dreptunghi. Analog
1 1
A C A C este dreptunghi.
Atunci segmentele [ ] [ ] [ ]
1 1 1
, , A A B B C C sunt congruente i se
njumtesc deci punctele
1 1 1
, , A B C , , , A B C sunt conciclice. Dreptele
1 1
B C i BC fiind paralele i
1 1
2
BH
A B A C = = deducem c
1 1
B C A A
este trapez isoscel, deci inscriptibil, deci A se afl pe cercul
circumscris triunghiului
1 1
, B C A aadar conciclic cu cele ase puncte
anterioare. Analog , B C se gsesc pe acelai cerc cu cele ase puncte
anterioare, deci cele nou puncte sunt conciclice.
Soluia 2. Fie
3 3 3
, , A B C punctele de intersecie ale nlimilor
, , AA BB CC cu cercul circumscris triunghiului ABC i
2 2 2
, , A B C
punctele diametral opuse punctelor , , A B C n acest cerc. Considerm
omotetia de centru H iraport
1
2
. Conform propoziiei 1 imaginile
punctelor
2 2 2
, , A B C prin aceast omotetie sunt punctele , , A B C ,
imaginile punctelor
3 3 3
, , A B C sunt punctele , , A B C , iar imaginile
punctelor , , A B C sunt punctele
1 1 1
, , A B C respectiv. Aadar punctele
, , A B C , , , A B C ,
1 1 1
, , A B C se afl pe un cerc, omoteticul
cercului circumscris triunghiului ABC prin omotetia de centru H i
raport
1
2
.
Observaie. Centrul acestui cerc este mijlocul segmentului [ ] OH .
Prezentm acum alte dou aplicaii care vizeaz conceptul de
ortocentru.
Problema 4. Se consider un triunghi ABC . Un cerc de diametru
[ ] EF cu [ ] EF [ ] BC este tangent dreptelor AB i AC respectiv n
punctele M i . N S se arate c dreapta MN trece prin ortocentrul
triunghiului . AEF
8
Soluie. Fie O centrul cercului, { } , , P AO MN AA BC =
{ } , A BC H AA MN = iU al doilea punct de intersecie al dreptei
AE cu cercul. Triunghiul ANO este dreptunghic n N i AO MN i
atunci conform teoremei catetei avem ( )
2
1 . AN AP AO = Cum
( ) . . APH AA O UU rezult c ( ) 2 . AP AO AH AA =
Conform puterii punctului fa de cerc rezult c
( )
2
3 . AN AU AE = Din ( ) ( ) 1 , 2 i ( ) 3 deducem c
AU AE AH AA = i atunci AHU AEA . . deci . HU AE Cum
FU AE rezult c , , F H U sunt coliniare i deci H este ortocentrul
triunghiului . AEF
Problema 5. Punctele planului se coloreaz cu 3 n culori date.
S se arate c putem gsi ntotdeauna un triunghi avnd ortocentrul
diferit colorat fa de fiecare din vrfurile triunghiului.
Soluie.Alegem punctele necoliniare , , A B C de culori diferite.
a)Dac triunghiul ABC nu este dreptunghic considerm H ortocentrul
su. Distingem cazurile:
1) culoarea lui H difer de culorile vrfurilor , , A B C ; n acest caz
problema este rezolvat.
2) H este la fel colorat ca unul dintre vrfuri (fie acesta A); cum C
este ortocentrul AHB i C nu are aceeai culoare cu , , A H B
problema este rezolvat.
b)Dac triunghiul ABC este dreptunghic (n A ) alegem M un punct
pe nlimea din A n interiorul triunghiului ABC . Dac M este
colorat diferit de B i C alegem triunghiul nedreptunghic MBC i
suntem n cazul a). Dac M are (de exemplu) culoarea lui B alegem
triunghiul MAC i suntem n cazul a).

n ncheiere propunem spre studiu urmtoarele aplicaii:

1). Se consider un cerc de diametru [ ] AB i M un punct interior
cercului, [ ] M AB . Fie N i P punctele de intersecie ale dreptelor
AM respectiv BM cu cercul i { } . C AP BN = Atunci . MC AB
2). Se consider triunghiul ascuitunghic ABC i H ortocentrul su.
Atunci ( ) 4 . AH BC BH AC CH AB Aria ABC + + =
w
w
.
n
e
u
t
r
i
n
o
.
r
o
9
3). Fie ABC un triunghi ascuitunghic cu
( ) ( ) ( ) ( ) ( ) , , , , 60 AA BC A BC BB AC B AC m ACB =

< i
( ) 30 . m BB A =

< Atunci triunghiul ABC este echilateral.
4). Fie M un punct interior unui triunghi ABC astfel nct simetricele
lui M fa de laturile triunghiului sunt situate pe cercul circumscris
triunghiului. Atunci M este ortocentrul triunghiului.
5). n triunghiul ascuitunghic ABC , fie O centrul cercului circumscris,
H ortocentrul triunghiului, iar M
mijlocul lui [ ] AH . Dac E este mijlocul lui [ ] BC i { } ME OH N =
atunci .
2
OC
MN =
6). Fie H ortocentrul triunghiului ascuitunghic ABC i D intersecia
dreptei AH cu cercul circumscris triunghiului. Artai c DH DB =
dac i numai dac ( ) 60 . m ACB =

<
7). Fie ptratul ABCD i punctele E i F n exteriorul ptratului astfel
nct triunghiurile ABE i BCF s fie echilaterale. Fie M mijlocul lui
[ ] DE i { } . H CE DB = Atunci , , , A M H F sunt coliniare.
8). Fie BB C C un ptrat ,iar ABC un triunghi construit n xeteriorul
ptratului. S se arate c perpendiculara din B pe AC , perpendiculara
din C pe AB i nlimea AA a triunghiului ABC sunt concurente.

Bibliografie:

1. Gheorghe ieica Probleme de geometrie, Editura Tehnic
Bucureti.
2. Colecia Gazeta Matematic.

Profesoar, Grupul colar Industrial Tismana, Gorj
Profesor, Colegiul Naional Ecaterina Teodoroiu,Gorj








10
Generalizarea unei probleme de concurs
Nicolae Stniloiu

La concursul Victor Vlcovici din anul acesta (2008) la clasa a X-a a
fost propus sub semntura domnului profesor Laureniu Panaitopol
urmtoarea problem:
1. Fie
3 2 1
z , z , z trei numere complexe care satisfac relaiile:
1 3 2 1
2 2 z z z z = +
2 1 3 2
2 2 z z z z = +
3 2 1 3
2 2 z z z z = +
S se arate c
3 2 1
z z z = = .
Citind aceast problem mi-am adus aminte c printre dosarele cu
probleme personale,cu ncercri i cutri proprii, exist de vreo 10 ani
urmtoarea problem:
Fie
3 2 1
z , z , z trei numere complexe care satisfac relaiile:
3 3 2 1
z z z z = +
1 1 3 2
z z z z = +
2 2 1 3
z z z z = +
S se arate c
3 2 1
z z z = = .
Tot printre acele multe hrtii, uitate cteodat, exist ns i urmtoarea
generalizare, care ni se pare demn de luat n seam:
G1. Fie
3 2 1
z , z , z trei numere complexe. Dac
3 2 1 3 2 1
, , , , ,
sunt numere reale astfel nct:
3 2 1 3 2 1
+ + = + + ,
3 2 3 1 2 1 3 2 3 1 2 1
+ + + + i
3 3 2 2 1 1 3 3 2 2 1 1
z z z z z z + + = + +
3 1 2 3 1 2 3 1 2 3 1 2
z z z z z z + + = + +
3 2 2 1 1 3 3 2 2 1 1 3
z z z z z z + + = + + . Atunci
3 2 1
z z z = = .
Vom da soluia acestei generalizri urmnd s prezentm apoi o alt
generalizare a acestei probleme.
Vom evalua expresia: ( )
2
3 3 2 2 1 1 3 2 1
, ,
3 2 1
z z z z z z E

+ + = .
w
w
.
n
e
u
t
r
i
n
o
.
r
o
11
Avem: ( ) ( )
( )
1 2 3
1 2 3 1 1 2 2 3 3 1 1 2 2 3 3
, , E z z z z z z z z z

= + + + + =
( ) ( ) ( )
2 2 2
2 2 2
1 1 2 2 3 3 1 2 1 2 1 2 1 3 1 3 1 3 2 3 2 3 2 3
z z z z z z z z z z z z z z z + + + + + + + +
Observm c egalitile din ipotez sunt echivalente cu:
( ) ( )
3 2 1 3 2 1
, , , ,
3 2 1 3 2 1
z z z E z z z E

= (1)
( ) ( )
3 2 1 3 2 1
, , , ,
1 3 2 1 3 2
z z z E z z z E

= (2)
( ) ( )
3 2 1 3 2 1
, , , ,
2 1 3 2 1 3
z z z E z z z E

= (3)
Din nsumarea relaiilor (1), (2) i (3) n form desfurat obinem:
( )( )+ + + + +
3 3 2 2 1 1
2
3
2
2
2
1
z z z z z z
( )( )
( )( )
( )( )
2 3 3 2 1 3 3 1 1 2 2 1 3 2 3 1 2 1
3 3 2 2 1 1
2
3
2
2
2
1
2 3 3 2 1 3 3 1 1 2 2 1 3 2 3 1 2 1
(4)
z z z z z z z z z z z z
z z z z z z
z z z z z z z z z z z z
+ + + + + + + +
+ + + + + =
+ + + + + + + +




Din egalitatea
3 2 1 3 2 1
+ + = + + , rezult:
( )
3 2 3 1 2 1 3 2 3 1 2 1
2
3
2
2
2
1
2
3
2
2
2
1
2 + + = + +
innd cont de relaia anterioar ,relaia (4) devine:
( )
2 3 3 2 1 3 3 1 1 2 2 1 3 3 2 2 1 1
2 z z z z z z z z z z z z z z z z z z + + sau:
0
2
1 3
2
3 2
2
2 1
= + + z z z z z z sau
3 2 1
z z z = = .
G2. Dac
n 2 1
z ..., , z , z sunt n numere complexe,
n n
..., , , , ..., , ,
2 1 2 1
sunt numere reale astfel nct:
n n
... ...
2 1 2 1
+ + + = + + + ,

< <

n j i
j i
n j i
j i
i
( )
( ) ( )
( ) ( )
( ) ( )

=
=
=
+ + + =

n n
n n
n n
n n n
z z z E z z z E
z z z E z z z E
z z z E z z z E
z z z z z z E
n n n n
n n
n n
n
,..., , ,..., ,
..... .......... .......... .......... .......... .......... .......... ..........
,..., , ,..., ,
,..., , ,..., ,
... ,..., ,
2 1 ... 2 1 ...
2 1 ... 2 1 ...
2 1 ... 2 1 ...
2 2 1 1 2 1 ...
1 2 1 1 2 1
1 3 2 1 3 2
2 1 2 1
2 1






atunci:
n
z z z = = = ...
2 1

Profesor, Boca

12
Metode de rezolvare a problemelor cu unghiuri n
spaiu
Maria Iancu
Unghiurile reprezint elementele fundamentale din geometria
spaiului euclidian i tematica unghiurilor propune probleme de mare
finee i dificultate.Din considerente de ordin metodic, voi aborda tema
dupa cum urmeaz:
A). Unghiul a dou drepte n spaiu (n general necoplanare);
B). Unghiul unei drepte cu un plan ;
C). Unghiuri diedre. Unghiul a dou plane.
n cadrul fiecarei seciuni voi prezenta diverse metode de aflare a
msurii unghiurilor, pe care le voi ilustra cu ajutorul unor probleme
rezolvate.
Sunt de prere c prin determinarea unui unghi trebuie s
ntelegem construirea lui, justificarea construciei fcute i apoi aflarea
msurii acelui unghi.
A. Unghiul a dou drepte n spaiu
Definiie 1. Fie d i g dou drepte n general necoplanare. Se numete
unghiul dreptelor d i g, unghiul ascuit(cel mult drept) format n orice
punct P al spaiului, prin ducerea de paralele la dreptele date.
Observaie 1. Unghiul a dou drepte d i g, nu este o figur geometric
unic determinat, dar msura unghiului celor dou drepte este un numr
bine precizat.
Definiie 2. Dou drepte d i g se numesc perpendiculare (ortogonale),
dac msura unghiului dintre ele are 90

.
Metode. Determinarea unghiului a dou drepte n spaiu poate fi
realizat prin metode specifice geometriei plane i prin metode proprii
geometriei spaiului care sunt consecine ale definiiei, dup cum
urmeaz :
M1). Dac dreptele d i g

se dovedesc a fi coplanare i neparalele,
unghiul lor se determin ncadrndu-le, de regul, ntr-un triunghi
i facnd apel la geometria triunghiului respectiv.
Exemplu .
1
) A Prisma dreapt regulat
/ / / /
ABCDA B C D are latura
bazei AB a =

i nlimea
/
2 AA a = . Aflai
/
MD , unde
( )
/
M BD
tiind c ( )
0
90 . m AMC = X
Soluie. Deoarece ( )
0
90 m AMC = X MAC dr.i n MAC :
A
'

w
w
.
n
e
u
t
r
i
n
o
.
r
o
13
AO OC = , unde { } AC BD O MO = este mediana coresounztoare
ipotenuzei
2
BD
MO OA OB = = = . n , MBD MO = median
i
2
BD
MO MBD = = dreptunghic cu ipotenuza BD
DM MB sau DM BD . Aplicnd Teorema catetei n
( )
2 2
4
2
1
D D a
D DB D D D M D B D M D M
D B D B

= = =

.
Din D DB 6 D B a = ; nlocuind n ( ) 1 avem :
2 6
3
a
D M = .
M2). Dac dreptele d i g nu sunt coplanare, printr-un punct ales
convenabil pe dreapta d, se duce o paralel h la dreapta g i apoi se
determin unghiul dintre d i h, care reprezint unghiul dintre
dreptele d i g.
O redactare corect a unei astfel de probleme presupune parcurgerea
unor etape:
(E1). Construcia unghiului (uneori este fcut deja !).
(E2). Justificarea construciei fcute.
(E3). Aflarea msurii acelui unghi.
Exemplu.
)
2
A Pe planul ptratului ABCD de latur a cm, se ridic de
aceeai parte a planului perpendicularele AA BB CC DD h = = = = .
Dac M este mijlocul segmentului ( ) AA i N mijlocul segmentului
( ) DD , iar unghiul dintre dreptele MD i NC

are 60

, artai c
2 h a = .
Soluie. ( ) NC DCC i ( ) MD ADD , iar dreapta comun celor
dou plane este DD .(E
1
) : Prin N , mijlocul lui ( ) DD construim
NA MD | .(E
2
) : Din A M ND | i A M ND = A NDM
paralelogram NA MD | ( ) ( ) ; ; MD NC NA NC A NC < < <
( ) 60 m A NC =

< .(E
3
) : ( ) . . NDA NDC CC NA NC =
NA C = isoscel i ( ) 60 m A NC =

< NA C = echilateral
A
B
B
C
D
N
B
'
C
'

D
'

14
2 NA A C a = = ( ) 1 . Aplicnd Teorema lui Pitagora pentru A N
n NDA
2
2 2
4
h
AN a = + ( ) 2 .Din ( ) 1 i ( ) 2 se obine 2 h a = .
M3). Dac dreptele d i g nu sunt coplanare, se ncadreaz cele
dou drepte n dou plane care au o dreapt comun h i printr-un
punct ales convenabil al acestei drepte h se duce o paralel
d
1
la dreapta d i o paralel g
1
la dreapta g i apoi se afl unghiul
dintre d
1
i g
1
care reprezint unghiul dintre dreptele d i g.
O redactare corect a unei astfel de probleme presupune parcurgerea
etapelor prezentate anterior.
)
3
A Pe planul ptratului ABCDde latur 2 AB a = cm se ridic,
de aceeai parte a planului, perpendicularele AA BB CC DD = = =
Dac A C C B , artai c ABCDA B C D este cub.
Soluie.Vom construi unghiul dintre dreptele necoplanare A C i C B
( ) A C ACC i ( ) C B BCC , iar ( ) ( ) ACC BCC CC =
(E
1
) : Fie M mijlocul lui ( ) CC .Prin M construim paralele la dreptele
BC i A C .(E
2
) : Fie { } A C B D O = i { } AC BD O = .

OM este linie mijlocie n A CC OM A C | .
Construim MN BC | , unde N mijloc ( ) BC ( ) ; A C C B <
( ) ; OM MN < OMN < ( ) m O MN < 90 =


(E
3
) : Cu teorema lui Pitagora, avem:
2
2 2
2
4
C C
MC O O M a

= +
( ) 1 , iar n MCN
2
2 2
4
C C
MN a

= + ( ) 2

.
Din O ON
2 2 2
ON OO ON = +
2 2 2
ON C C a = + ( ) 3 .
Deoarece OMN este dreptunghic
2 2 2
ON OM MN = + . Din
( ) 1 , ( ) 2 , ( ) 3 , obinem :
2 2
2 2 2 2
2
4 4
C C C C
C C a a a

+ = + + +

2 C C a = ABCDA B C D cub.
(Va urma)

Profesor, coala Romul Ladea, Oravia
'
w
w
.
n
e
u
t
r
i
n
o
.
r
o
15
Diversitate n curriculumul colar-opional de
matematic
Tudor Deaconu
Curriculum-ul elaborat n coal, opionalul, ofer
posibilitatea manifestrii creativitii cadrului didactic, permindu-
i acestuia s-i conceap obiective, coninuturi, strategii didactice
proprii, experimente, care s rspund problematicii diversitii n
curriculum. n cadrul orelor de la clas, ct i n cadrul activitilor
desfurate n cadrul Centrului de excelen am cutat s-i
mobilizez i s-i motivez pe elevii mei spre a ndrgi matematica.
i pentru c ei ndrgesc matematica, iar prinii lor au optat
pentru aceast disciplin, am inclus n schema orar opionalul
CURIOZITI MATEMATICE la clasa a V-a.
Proiectul curricular l-am ntocmit pentru ntregul an colar,
stipulnd n rubrica observaii c, pe parcursul derulrii n timp a
activitii didactice, el poate suferi modificri generate de
evoluia/involuia rezultatelor obinute de elevi, dar i pe baza
experimentelor aplicate i a rezultatelor acestora.
Organizarea tematic a orizontului referenial am realizat-o
respectnd structura i formatul de proiectare al unui opional la
nivelul disciplinei care cuprinde: obiective cadru, obiective de
referin, activiti de nvare, evaluare, list de coninuturi. n
lipsa unui model care s fundamenteze desfurarea unui opional,
am realizat proiectarea pornind de la obiectivele-cadru ale
disciplinei matematic, din care am dedus aplicaiile practice,
experimentele i obiectivele de referin, dup cum urmeaz:
I.Cunoaterea i utilizarea conceptelor specifice matematicii:
1. S utilizeze conceptele specifice matematicii n scopul
obinerii de informaii noi;
2. S analizeze i s fac conexiuni logice ntre informaii;
3. S descopere, s recunoasc i s utilizeze corespondene
simple i succesiuni de numere asociate dup criterii date.
II.Dezvoltarea capacitilor de explorare/investigare i rezolvare a
problemelor:
4. S identifice problemele din contextul n care acestea apar n
via sau n diverse segmente ale cunoaterii;
5. S descopere tehnici adecvate de rezolvare a problemelor;
6. S modeleze matematic, pe baza analizei logice, probleme
16
7. enunate discursiv i s le rezolve;
8. S exploreze modaliti practice de compunere/descompunere
a ptratului Tangram, utiliznd tanurile;
9. S colecteze date i s le prelucreze dup criterii date.
III.Formarea i dezvoltarea capacitii de comunicare, utiliznd
limbajul matematic:
10. S descrie clar i concis demersurile ntreprinse n
descoperirea soluiilor;
11. S obin informaiile dorite, adresnd ntrebri incisive i
pertinente, care pot fi probate i la care se pot da rspunsuri
precise.
IV. Dezvoltarea interesului i a motivaiei pentru studiul i aplicarea
matematicii n contexte variate:
1. S manifeste un comportament adecvat, n relaiile cu colegii
dintr-un grup de lucru;
2. S manifeste disponibilitate pentru a nva de la alii i
pentru a-i ajuta pe ceilali s nvee;
3. S manifeste omogenitate atitudinal.
Prin obiectivele menionate mai sus, am stabilit coninuturile
nvrii, plecnd de la premisa c ora de opional nu trebuie s se
confunde/ suprapun cu cea de matematic. Consider c opionalul
poate deveni un mod eficient de a trezi interesul i curiozitatea
elevilor pentru aceast disciplin, experimentnd ceva cunoscut n
alte situaii date i nici de cum o aprofundare sau extindere a
programei i manualului. Eficiena maxim a orei de opional a fost
asigurat de convergena activitilor didactice cu cea mai liber
form de instruire care este jocul i experimentul. Eventualele
fenomene anomice, care ar fi putut surveni din cauza jocului
matematic, au fost prevenite prin transformarea nvrii prin joc
ntr-un proces constructiv de experimentare, n centrul cruia s-a
aflat tot timpul elevul. Am constatat c acesta nva modelul, i-l
fixeaz exersnd, experimenteaz pe alte exemple, descoper
soluiile i transfer procedeele de lucru.
Nu trebuie uitat faptul c matematica este o disciplin bazat pe
reflecie, iar elevul nu trebuie lsat s se limiteze la contemplarea
situaiei n care a fost pus. El reflecteaz asupra problemelor
propuse, imagineaz soluii, experimenteaz, i confrunt opiniile
cu cele ale colegilor si, corecteaz eventualele erori.

w
w
.
n
e
u
t
r
i
n
o
.
r
o
17
Coninuturile nvrii:
1. Operaii cu numere naturale
2. Operaii cu rezultate codificate
3. Ptrate perfecte
4. Ptratele magice: ptratul Tangram
5. Curioziti numerice: compararea puterilor
6. irul numerelor naturale: operaii i operatori
7. Procedee de calcul rapid al unor sume de mai muli termeni
8. Funcii numerice: probleme de numrare
9. Principiul cutiei (Dirichlet)
10. La grania dintre matematic i logic: scurte proze cu tlc
matematic
11. Mulimi finite i mulimi infinite: mulimea divizorilor i mulimea
multiplilor unui numr natural
12. Operaii cu mulimi de numere
13. Numere ntregi
14. Numere raionale
15. Operaii cu numere raionale
16. Periodicitate
17. Aproximri zecimale
18. Localizarea n plan a unui punct de coordonate ntregi
19. Uniti de msur
20. Transformri.
Tipuri de activiti de nvare
- exerciii-joc pe tema operaiilor cu numere naturale;
- alegerea drumului unui labirint cu mai multe alternative de ieire;
- stabilirea unei corespondene ntre un set de exerciii i codul ce
conduce la obinerea unei secvene literale;
- exerciii de determinare a numerelor dintr-un careu, respectnd
constanta ptratului;
- exerciii de descoperire a unor proprieti mai ciudate ale
numerelor: 9, 37, 45, 100;
- invenii de reguli pentru compunerea irurilor; completarea unor
secvene numerice date descoperind criteriul;
- exerciii de calculare a unei sume de mai muli termeni; deducerea
algoritmului i aplicarea lui pe segmente din irul numerelor naturale;
- tabele, diagrame i grafice;
- exerciii de ghicire a regulei;
- determinarea elementelor din domeniu/co-domeniu, cnd se cunosc
elementele co-domeniului/domeniului i regula de coresponden;
18
- exerciii-joc de compunere a unor imagini figurale, prin folosirea
tanurilor;
- transpunerea unor situaii problem n limbaj matematic; rezolvarea
lor;
- alctuirea de tabele cu dou intrri i aflarea soluiei;
- exerciii de identificarea problemei din contextul n care aceasta apare
i rezolvarea ei;
- exerciii de modelare matematic, pe baza analizei logice, a
problemelor enunate discursiv i rezolvarea lor;
- transpunerea situaiilor-problem n limbaj matematic; rezolvarea lor.
Concluzii
n centrul complexului de obiective pe care le implic activitile
cu coninut matematic, rezolvarea problemelor rmne o activitate de
profunzime cu caracter de analiz i sintez superioare. Rezolvarea de
probleme pune la ncercare, n cel mai nalt grad capacitile intelectuale
ale copiilor, le solicit toate disponibilitile psihice, n general
inteligena.
n activitatea de rezolvare de probleme care a stat la baza
coninuturilor nvrii, o atenie deosebit am acordat nelegerii datelor
acestora, pentru a-l ajuta pe elev s construiasc judecile i
raionamentul care conduce la rezolvarea lor. Primele probleme introduse
n activitile matematice au fost cele a cror rezolvare s-a realizat la un
nivel concret cu date i aciuni din viaa de zi cu zi. La baza gsirii
soluiei acestor probleme au stat intuiiile secundare, elementele
componente ale inteligenelor multiple, achiziiile cognitive constituite n
procesul de educaie tiinific, experimentul asupra unor activiti cu
coninut matematic. Aceste probleme rezolvate cu elevii au avut caracter
practic concret, aciunea fiind ilustrat prin experimente executate de
copii, crora le-am oferit un bogat material demonstrativ n contextul
diversitii n curriculum.
Se tie c efortul pe care l face copilul n rezolvarea contient a
problemelor presupune o mare mobilitate a proceselor psihice cognitive.
Cea mai solicitat i antrenat este gndirea, prin operaiile logice de
analiz, sintez, comparaie, abstractizare i generalizare. Prin
descoperirea cii de rezolvare a problemelor se sporete flexibilitatea
gndirii, se educ perspicacitatea, spiritul de iniiativ, lucrul n echip i
ncrederea n forele proprii n condiiile n care diversitatea devine o
component a vieii noastre cotidiene.

Profesor, Director CCD Cara-Severin
w
w
.
n
e
u
t
r
i
n
o
.
r
o
19
Calculul unor sume cu combinri
Nicolae Stniloiu
Calculele duse cu puin efort i mare atenie pentru soluionarea
problemei X.113 din RMCS nr.24, prezentate n acest numr al revistei,
au condus la redactarea acestei note. Sunt bine cunoscute egaliti ca:
1)
0
2 ,
n
i n
n
i
C n
=
=

N . 2)
1
1
2

=
=

n
n
i
i
n
n iC , , 1 n n N .
3) ( ) ( )
2
2
2 1 1

=
=

n
n
i
i
n
n n C i i , , 2 n n N . Vom da o metod de
calcul pentru sume mai generale, de forma: ( )

=
n
i
i
n
C i P
0
, unde
( ) . grad P p = Mai nti vom enuna un rezultat fr s-l mai demonstrm.
Fie polinoamele ( ) ( ) ( ) ( ) ( )
0
1, 1 ... 1 , 1
i
Q x Q x x x x i i = = + .
Propoziia 1: Dac P este un polinom de grad pN , atunci exist
constantele reale
i
astfel nct:
0
p
i i
i
P Q
=
=

.
Obs: Propoziia precedent exprim faptul c polinoamele , 0,
i
Q i p =
este o baz pentru spaiul vectorial al polinoamelor de grad cel mult p.
Propoziia 2: ( ) ( )
k n
k
i
n
n
i
k
n Q C i Q

=
=

2
0
, pentru orice nN.
Demonstraie: Se observ c: ( ) ( )
k i
k n k
i
n k
C n Q C i Q

= i atunci:
( ) ( ) ( )
k n
k
n
i
k i
k n k
i
n
n
i
k
n Q C n Q C i Q

=

=
= =

2
0 0
.
Observaie: n sumele anterioare combinrile care apar cu indice
superior negativ le considerm nule.
Propoziia 3: Dac P este un polinom de grad p atunci exist un
polinom F de grad cel mult p astfel nct: ( ) ( )
p n
n
i
i
n
n F C i P

=
=

2
0

Demonstraie: Din propoziia 1 exist
j
astfel nct: ( ) ( )

=
=
p
j
j j
i Q i P
0

i atunci: ( ) ( ) ( )
0 0 0 0 0
p p n n n
i i i
n j j n j j n
i i j j i
P i C Q i C Q i C
= = = = =

= = =




20

( ) ( ) ( ) ( ) ( )
0 0 0
2 2 2 2 ,
p p p
n j n p p j p j
j j j j j j
j j j
Q n Q n F x Q x gr F p

= = =
= = =


Propoziia 3 ne d dreptul s elaborm urmtoarea metod de calcul
al sumelor de tipul: ( )

=
n
i
i
n
C i P
0
unde ( ) grad P p = .
Se ncearc determinarea polinomului F de grad p astfel
nct ( ) ( )
p n
n
i
i
n
n F C i P

=
=

2
0
lund pentru n suficiente valori.
Problema de care aminteam la nceput (sursa cutrilor noastre)
cere calculul sumei:

=
n
i
i
n
C i
1
3
. Evident ( )
3
i i P = i ( ) 3 grad P = .
Presupunem ( ) d cx bx ax x F + + + =
2 3
.
n egalitatea: ( )
3
0
3
2

=
=

n
n
i
i
n
n F C i , dm lui n valorile 3, 4, 5 i 6 i
obinem:

= + + +
= + + +
= + + +
= + + +
324 6 6 6
200 5 5 5
112 4 4 4
54 3 3 3
2 3
2 3
2 3
2 3
d c b a
d c b a
d c b a
d c b a

Determinantul sistemului de mai sus este de tip Vandermonde iar
rezolvarea acestuia nu ar trebui sa fie o problem. Se obine
1, 3, 0, 0 a b c d = = = = i deci: ( )
2 3 3
0
3
3 2 n n C i
n
n
i
i
n
+ =

=


Ca metod alternativ pentru calculul sumei

=
n
i
i
n
C i
0
3
amintim o
metod la nivelul clasei XI-a: dezvoltarea ( )
n
n
i
i
n
i
x C x + =

=
1
0
se
deriveaz o data n raport cu x iar egalitatea obinut se nmuleste cu x,
apoi se repet procedeul de dou ori i n final se ia 1. x =
Profesor, Boca


w
w
.
n
e
u
t
r
i
n
o
.
r
o
21
Partea ntreag, partea fracionar a unui numr real
Lucian Dragomir
I. Definiii, notaii, proprieti.

Dac un numr real a are scrierea zecimal
0 1 2 3
, ... a a a a a = (evident
0
a Z i { }
1 2 3
, , ,... 0,1, 2,..., 9 a a a ), atunci partea ntreag a lui a se
noteaz cu [ ] a i se definete prin [ ]
0
0
, dac 0
1 , dac 0
a a
a
a a

=

<

.
Mai simplu de reinut: partea ntreag a lui a este cel mai mare numr
ntreg care nu l depete pe a (cel mult egal cu a); dac facem apel la
reprezentarea pe ax a numerelor reale, atunci [ ] a este primul numr
ntreg din stnga lui a .
n baza axiomei lui Arhimede, se poate spune i: pentru orice numr
real a , exist un unic numr ntreg, notat cu [ ] a , astfel nct
[ ] [ ] 1 a a a < + ; aadar,pentru orice aR avem: [ ] a k = Z dac i
numai dac 1. k a k < +
Prin definiie, partea fracionar a numrului real a este { } [ ] a a a = ;
evident, { } [ ) 0,1 a .
Exemple:
[ ] [ ] [ ] 1, 25 1; 7,89 7; 3, 28 4; 3 1; 5 3;

= = = = =

{ } { }
7
0, 5; 7 0; 3, 45 0,55.
2

= = =



Vom trece n revist cteva proprieti i rezultate utile, des folosite n
rezolvarea problemelor:
P1. [ ] 1 , . a a a a < R
P2. [ ] , , . a k a k a k < < R Z
P3. [ ] [ ) , , 0,1 . k k k + = Z
P4. [ ] [ ] , , . a k a k a k + = + R Z
P5. { } { }, , . a k a a k + = R Z
P6. [ ] [ ] { } { } { } { } [ ] { }
, , 0, . a a a a a a a = = = =

R
22
P7. Dac , a bRi [ ] [ ] a b = , atunci 1. a b < ( Reciproca este fals; e
suficient un contraexemplu:
1 1
,
3 3
a b = = )
P8. { } { } ( ) , , . a b a b a b = Z R
P9. [ ] [ ] [ ] [ ] [ ] 1, , . a b a b a b a b + + + + R
P10. [ ] [ ]
1
2 , .
2
a a a a

+ + =


R (identitatea lui Hermite; de remarcat c
aceasta admite o generalizare destul de des utilizat:
[ ] [ ]
1 2 1
... , , .
n
a a a a na a n
n n n


+ + + + + + + =


R N )
II. Probleme rezolvate.
1. S se arate c [ ] [ ], na nb n = N dac i numai dac . a b =
(Gh.Andrei, OJ Constana, 1996)
Soluie: Dac a b = prima egalitate este evident. Presupunnd c a b i
[ ] [ ], na nb n = N ,obinem
{ } { } { } { } ( ) ( ) 1,1 na na nb nb n a b na nb = = , n N. Dac
a b > , membrul drept al ultimei egaliti poate fi orict de mare,
contradicie. Analog dac a b < , aadar . a b =
2. Dac , a bR,s se arate c { } { } 1 a b + = dac i numai dac
( ) a b + Z i , \ . a bR Z
(Gh.Andrei)
Soluie: Dac , \ a bR Zi ( ) a b + Z,atunci { } { } [ ) 0, 2 ; a b + cum
{ } { } [ ] [ ] a b a a b b + = + Z,deducem c { } { } { } 0,1 . a b + Deoarece
{ } { } 0 a b + (n caz contrar am ajunge la , a bZ, contradicie), obinem
{ } { } 1 a b + = .Reciproc, dac { } { } 1 a b + = , avem { } { } 0, 0 a b i,din
{ } { } [ ] [ ] 1 a b a a b b = + = + , deducem ( ) a b + Z.
3. a) S se dea un exemplu de numr real t pentru care { } { } 2 1. t t + >
b) S se rezolve ecuaia
4 1 3 1
.
3 4
x x +
=



w
w
.
n
e
u
t
r
i
n
o
.
r
o
23
Soluie: a) De exemplu,
7
5
t = (verificare imediat); b) dac
3 1
4
x
k
+
= Z, atunci avem
4 1
(1)
3
k
x

= .Pe de alt parte,din
3 1 4 1 3 1
1
4 3 4
x x x + +
< + se ajunge imediat la
19
1, .
7
x


Folosind acum
(1), deducem
4 1 19
1
3 7
k
< ,de unde
16
1
7
k < ; cum ns k Z,
obinem { } 1, 2 k i, corespunztor,
7
1, .
3
x




4. a) S se dea un exemplu de numr raional ( ) 1, a pentru care
0.
3 5
a a
+ =



b) S se determine partea ntreag a numrului 3 5. b = +
c) S se rezolve ecuaia
1 1
.
3 5
x x +
=



Soluie: a) Dac 3 a ,egalitatea nu este posibil;cutm astfel ( ) 1, 3 a ;
e suficient de exemplu,
15
.
14
a = b) Aproximri simple conduc la posibilul
rezultat [ ] 3 b = (numrul b pare fi aproape 4, sau chiar l depete?);
ncercm astfel s artm c 3 4 b < < ; prin ridicare la ptrat, aceasta este
echivalent cu :
9 8 2 15 16 1 2 15 8 1 60 64 < + < < < < < , evident adevrat, aadar
[ ] 3 b = ; c) evident,
1
5
x
k

= Z conduce la 5 1 (1). x k = + Z Din


1 1 1
1
5 3 5
x x x +
< + ajungem imediat la
7
4,
2
x



i, innd cont de
(1), obinem { } 4,1 x .
5. S se determine numerele reale pozitive x i y pentru care
[ ]
[ ]
2 1,5
2 2,5
x y
y x
+ =

+ =


Soluie: Adunm ecuaiile sistemului dat i ajungem la
[ ] [ ] { } { } 3 3 4 x y x y + + + = ; deoarece , 0, , x y x y Z i
24
{ } { } [ ) 0, 2 x y + , deducem c { } { } 1 x y + = i astfel avem i [ ] [ ] 1 x y + = .
Cazul [ ] [ ] 0, 1 x y = = este exclus datorit primei ecuaii, aadar rmne
doar [ ] [ ] 1, 0 x y = = , de unde obinem imediat 1,5 x = i 0, 5 y = .
6. S se calculeze partea ntreag a numrului
2 2 2
1 1 1
1 ...
2 3 2008
A = + + + + .
Soluie: Aceast sum ar trebui s fie o obinuit a casei pentru cei de
clasa a VI-a chiar; se folosete inegalitatea
2
( 1) k k k > , adevrat
, 2 k k N , aadar
2
1 1 1 1
, 2
( 1) 1
k
k k k k
k
< =

. Dm valori
acum lui k, anume 2,3,..., 2008 i nsumm, membru cu membru,
inegalitile obinute, apoi adunm(tot n fiecare membru) 1; vom ajunge
imediat la:
1
1 2 2
2008
A < < < . Evident, [ ] 1. A =
7. a) S se arate c nu exist numere reale x pentru care
2
1.
2
x
x

= +



b) S se determine numerele reale y pentru care
2
.
2
y
y

=



(Lucian Dragomir)
Soluie: a) Dac 0 x < , membrul stng este negativ, iar cel drept strict
pozitiv; evident, 0 x = nu verific, iar dac ( ) 0, 2 x , membrul stng este
0, pe cnd cel drept este supraunitar; n general acum, dac
[ ) 2 , 2 2 , x k k k

+ N , avem
2
x
k

=


i
2 2
1 4 , x k k k

+ > N ,
aadar ecuaia nu are soluii reale; b) se observ soluia 0 y = i, printr-un
raionament asemntor cu cel anterior, se arat c nu mai exist alte
numere reale y care s satisfac egalitatea din enun.
8. S se gseasc perechile ordonate ( ) , x y de numere pozitive i
distincte pentru care 1,5 x y + = i [ ] [ ] 0. x y y x + =
(Lucian Dragomir)
Soluie: A doua egalitate din enun se poate scrie
{ } { } 0 x y x y y x y x + = , de unde { } { } 0 x y y x = = .
Folosind P.8.avem c ( ) 2( ) x y y x x y + = Z, deci
w
w
.
n
e
u
t
r
i
n
o
.
r
o
25
,
2
k
x y k = Z . Deoarece avem i 1, 5 x y + = , deducem imediat c
3 3
,
4 4
k k
x y
+
= = . Cum ns , 0 x y ,ajungem la { } 0,1, 2,3 k .
Analizm uor aceste cazuri i obinem perechile cerute:
1 5 1 3
1, , , , , 0 .
2 4 4 2




9. S se determine partea ntreag a numrului
2
3 , . n n n

+ N
(Concurs Florica T.Cmpan,2003)
Soluie: Deoarece
2 2 2
2 1 3 4 4, n n n n n n n

+ + + < + + N , deducem
c
2
1 3 2 n n n n + + < + i astfel
2
3 1, . n n n n

+ = +


N
10. S se arate c partea fracionar a numrului
2
4 , n n n

+ N ,este
mai mic dect 0, 25.
(Concurs Unirea,2003)
Soluie: Deoarece
2
2 4 2 1, n n n n n

< + < + N , avem c
2
4 2 n n n

+ =


i deci
{ }
2 2
4 4 2 . n n n n n + = + Rmne de artat
c
2
4 2 0, 25 n n n + < , ceea ce este echivalent cu
2 2 2
4 0, 25 2 4 4 0, 0625 n n n n n n n + < + + < + + , ultima inegalitate
fiind evident adevrat.
11. S se rezolve ecuaia [ ] { }
2003 2002 2
... 1. x x x x x

+ + + + =


(Concurs Ucraina,2003)
Soluie: Membrul stng al ecuaiei este numr ntreg,deci , dac x este o
soluie a ecuaiei, avem { } x Z ;cum ns { } [ ) 0,1 x , ajungem la
{ } 0 x x = Z.Ecuaia devine astfel
2003 2002 2
... 1 0 x x x x + + + + + =
sau
( )
2002 2000 2
( 1) ... 1 0. x x x x + + + + + = Deoarece suma din a doua
parantez este strict pozitiv, se obine unica soluie a ecuaiei 1. x =
12. S se calculeze suma
2 3 3 4 1
1 2 ... ,
2 3
n n
S n
n


+ + + +

= + + + + +



N .
(Concurs Reghin,2005)
26
Soluie: Folosind P.4., avem imediat
3 4 1
1 2 1 1 ... 1
2 3
n
S
n

+

= + + + + + + + +



; deoarece
1
0 1, , 3
k
k k
k
+
< < N (inegalitate imediat), ajungem
la 1 1 1 1. S n n = + + = +
13. Dac , , , a b c d sunt patru numere naturale consecutive nenule,s se
arate c
.
2
a b c d
a b c d

+ + +

= + + +




(Dana Piciu,Concurs Gh.ieica,2006)
Soluie: Considerm , 1, 2, 3, a n b n c n d n n

= = + = + = + N i notm
1 2 3
2
n n n n
x
+ + + + + +
= , 4 6 y n = + .Prin ridicri la ptrat se
arat imediat c 4 5 3 4 6 n n n n + + + < + i
4 5 1 2 4 6 n n n n + + + + < + ,de unde ajungem la
4 5 4 6 n x n y + < + = ;deducem astfel c
[ ] [ ] 4 5 4 6 n x n y

+ + =

. Deoarece 4 6 n + nu este ptrat
perfect(ptratele perfecte sunt de forma 4m sau 8 1 p + ), rezult
4 5 4 6 n n

+ = +

, adic [ ] [ ]. x y =
14. S se calculeze suma
2 5 3 6 4 7 ... 100 103 S

= + + + +

.
(OL Arad,2006)
Soluie: Prin ridicri la ptrat se arat c 1 ( 3) 2, n n n n n

+ + < + N
i deci ( 3) 1, n n n n


+ = +

N . Folosind acest rezultat, deducem
imediat c 3 4 5 ... 101 5148. S = + + + + =
15. a)S se arate c [ ] [ ], , . n x nx x n
+
R N
b) S se determine toate numerele reale pozitive x pentru care
[ ] [ ] [ ] , 2 , 2 x x x sunt numere naturale consecutive.
(Mircea Fianu,OL Bucureti,2002)
Soluie: a) [ ] [ ] { } [ ] { } [ ]. nx n x n x n x n x n x = + = +


w
w
.
n
e
u
t
r
i
n
o
.
r
o
27
b) Pentru 0 x > avem aadar [ ] [ ] [ ] 2 2 x x x ;deoarece
[ ] [ ] { } 2 2 0,1 x x , se impune s avem [ ] [ ] 2 2 1 x x = i astfel ajungem
la { } 2 1 x =

, de unde
1
,1
2
x


. Verificm acum c toate aceste numere
satisfac condiia din enun.
16. S se arate c prin mprirea lui k Zla n

N se obine ctul
k
n



i
restul .
k
n
n




Soluie: Notm ctul i restul cu q , respectiv r i avem:
, 0 k nq r cu r n = + < . Deducem acum: 1
nq r k nq n
q q
n n n
+ +
= < = + ,
aadar .
k
q
n

=


Pe de alt parte, .
k k k nq r
n n n q r
n n n n
+
= = =





Observaie: Aceast problem arat faptul c dac n

N , atunci n
mulimea { } 1, 2, 3,..., k avem exact
k
n



multipli de n .
17. S se determine cte numere naturale nenule,mai mici dect 100,nu
sunt divizibile nici cu 2,nici cu 3.
Soluie: Ne referim aadar la mulimea { } 1, 2,3,...,99 n care avem
99
49
2

=


de numere divizibile cu 2 i
99
33
3

=


de numere divizibile cu
3. Am fi poate tentai s dm rezultatul 99 (49 33) + , ns trebuie s
remarcm c unele numere au fost numrate de dou ori (de exemplu 6,
12 , 18 , ) numrul acestora este egal cu
99
16.
2 3

=


Abia acum
putem da rezultatul corect: avem 99 (49 33 16) 33 + = de numere cu
proprietatea din enun.
18. S se determine care este exponentul lui 2 n descompunerea n
factori primi ai numrului 100! 1 2 3 ... 100. =
Soluie: n mulimea { } 1, 2,3,...,100 avem
100
50
2

=


de numere divizibile
cu 2; dintre aceste 50 de numere, fiecare al doilea este divizibil cel puin
28
cu puterea a doua a lui 2 sunt
2
100
25
2

=


de astfel de numere; dintre
acestea, fiecare al doilea este divizibil cel puin cu
3
2 . Raionm la fel n
continuare, inem cont c
7
2 100 > i c fiecare factor al lui 100! care
este divizibil cu 2
k
, dar nu i cu { } ( )
1
2 , 1, 2, 3, 4,5, 6,
k
k
+
,se socotete,
n modul indicat, de k ori ca fiind divizibil cu
2 3
2, 2 , 2 ,..., 2 .
k
Exponentul
cutat este astfel
2 6
100 100 100
... 50 25 12 6 3 1 97.
2
2 2

+ + + = + + + + + =



Observaie: n general, exponentul numrului prim p din descompunerea
n factori primi ai numrului ! n este
2 3
....
n n n
p
p p


+ + +




.
19. S se arate c pentru orice xReste adevrat egalitatea:

+
=

+
+

+
3
1
2
1
6
5
6
4
6
3 x x x x x
.
(Cristinel Mortici, OJ 2002)
Soluie: Notm
1
6
x
y
+
= i egalitatea propus devine
[ ] [ ]
1 1 2
3 2
3 2 3
y y y y y

+ + + + =


;aceasta se obine din identitile
de tip P.10.: [ ] [ ]
1
2
2
y y y

= + +


i [ ] [ ]
1 2
3
3 3
y y y y

= + + + +


.
20. Se consider numerele , ,1 p q q p

N i
( )
2
2
. a p q p = + + S
se arate c a este iraional i are partea fracionar strict mai mare
dect 0, 75.
(Adrian P.Ghioca,ON 2000)
Soluie: Din 1 q p rezult c
2 2 2 2
1
4
p p q p p p p < + + < + + i
astfel
2
1
2
p p q p < + < + . Deducem astfel c
2
\ p q + R .
Deoarece
2 2
2 2 a p q p p q = + + + i 1 p , obinem i c \ aR .
w
w
.
n
e
u
t
r
i
n
o
.
r
o
29
Considerm acum
( )
2
2
. b p q p = + Deoarece
( )
2
1
0
2
p q p < + < ,
rezult c
1
4
b < . S observm acum c
( )
2
2 2 a b p q c

+ = + = N ,
aadar
1
4
c c b a c < = < , de unde [ ] 1 a c = i deci
{ } [ ]
3
1 1 .
4
a a a c b c b = = + = >
III. Probleme propuse.
1. S se rezolve ecuaiile:
a)
2 2
.
3 4
x x +
=


b)
1 2 3
3 .
2 3 4
x x x
x
+ + +
+ + =



2. S se determine xRpentru care
{ }
[ ]
.
x
x
x
=
(Titu Andreescu, OL Sibiu, 2003)
3. S se determine [ ) , 0, x y tiind c ( ) x y + Ni
[ ] [ ]
2 2
2 3
12, 5
x y
x y
+ =

+ =

.
4. S se determine numerele naturale n pentru care 3 1. n n

= +


(Bulletin Math, Canada)
5. S se arate c
1
, .
2
n n n
n n


+ =


+ +

N
(Gheorghe Eckstein)
6. S se arate c
[ ]
[ ] 5
2 , , 1.
2
x
x
x x
x x
+ < R
(Liliana Niculescu)
7. S se determine n

N pentru care numrul
3 2
8 1
3
n n
n

+ +



este
prim.
(Gabriel Popa, Concurs Unirea,2003)
8. S se arate c, dac , , , a b x y sunt numere reale astfel nct
[ ] [ ] ax by bx ay + = + ,atunci
{ }
min , 1. a b x y <
(Traian Du, OL Braov,2006)
30
9. S se determine , x y Rpentru care
[ ] { }
{ } [ ]
2
2
x y
x y

.
(Concurs GM, 2006)
10. S se rezolve ecuaia
1 2 1 1
3 3 2 2
x x x x + + +
+ = +


.
(Drago Ungura, Concurs RMCS,2007)
Profesor, Oelu-Rou
Ultimele zile de primvar cu primele mari bucurii
matematice
Loreta Ciulu,Miruna Dalila Ciulu
Judeul nostru a fost reprezentat, la etapa naional a olimpiadei
celor mai mici, de o echip format din 4 elevi, nsoit de doamna
profesoar Mariana Drghici de la coala nr. 2 Reia. Cei care au reuit
s ajung n acest an pn acolo sunt: la clasa a V-a: Andrei tefnescu-
Oelu-Rou(coala Gen. 1), Ciulu Miruna- Reia(coala Gen.6), iar la
clasa a VI-a: Adina eudan Reia (coala 2), Silviu Lazr Reia
(coala 9). De fapt, numele erau cunoscute de la diverse concursuri
anterioare(ceea ce nu exclude absolut deloc ce poate urma: unii dintre
colegii lor s aib cel puin aceleai performane n anii care urmeaz)
Concursul s-a desfurat la coala TAKE IONESCU din
Rmnicu - Vlcea, n perioada 23-25 mai 2008, iar masa i cazarea au fost
la Colegiul Tehnic Forestier. Trebuie s remarcm faptul c din partea
judeului nostru, doamna RODICA IATAN a participat n calitate de
membru n Comisia de Evaluare i Organizare a Concursului.
Elevii de clasa a VI-a au obinut rezultate efectiv onorabile, chiar
fr a fi premiai; cei de clasa a V-a au fost chiar i premiai: Andrei a
obinut Meniune i medalie de bronz, iar Miruna a obinut Premiul I i
medalia de Aur...
Pentru a sublinia importana acestui concurs, trebuie s amintim
doar c premiile Ministerului au fost acordate chiar de ctre Domnul
Profesor Cristian Alexandrescu preedintele concursului, inspectorul
general de matematic din minister, iar medaliile au fost nmnate de
ctre Domnul Profesor Universitar Dr. Radu Gologan Preedintele
S.S.M.R.
Profesor, c. cu cls. I-VIII nr. 2 Reia

w
w
.
n
e
u
t
r
i
n
o
.
r
o
31
n pitorescul ora Rmnicu Vlcea, la coala Take Ionescu,
coala cu Ceas cum i se mai spune, am trit experiena deosebit pe
care i-o ofer participarea la un concurs naional. Competiia a fost
dificil, s-au ntrecut 103 concureni la clasa a V-a, subiectele grele,
concentrarea maxim. M-am strduit s fac o lucrare ct mai bun i am
reuit.. De aceea la afiarea rezultatelor m-am simit uurat i
mulumit. Doar cteva lacrimi trdau starea prin care treceam. Eram
emoionat, obosit,dar fericit.
Abia a doua zi la festivitatea de premiere cnd mi s-a acordat
Premiul I din partea M.E.C.T. i Medalia de aur din partea S.S.M.R am
neles ce performan am realizat. Sunt mndr c am reuit s aduc
medalia de aur acas, pentru mine, pentru coala mea, pentru judeul meu.
Rezultatul obinut este rodul unei munci continue, zi de zi, fcut
cu bucurie; pasiunea mea pentru matematic e susinut de ambiie,de
dorina mea de a nvinge i de ncrederea cu care abordez fiecare
problem. Mereu mi-am spus c pot, c trebuie s lucrez mai mult.
Sunt recunosctoare i i mulumesc pe aceast cale d-nei. prof.
Susana Simulescu, diriginta mea, care a fost alturi de mine n pregtirea
intens de la coal, dar nu n ultimul rnd susinerea i ajutorul din partea
prinilor mei sunt elemente eseniale n ecuaia succesului meu.
Elev, c. cu cls.I-VIII Nr.6 Reia


Graficul olimpiadelor i concursusurilor colare

Etapa pe coal a olimpiadei: ianuarie 2009(subiecte propuse de
comisia din coal)
Etapa local a olimpiadei: 14 februarie 2009(subiecte propuse de
Comisia judeean)
Ediia a IV a a Concursului RMCS, Oelu-Rou: 28 februarie 2009
Etapa judeean a olimpiadei: 7 martie 2009(subiecte propuse de
Comisia Central)
Etapa naional a Olimpiadei, clasele VII XII: 12-16 aprilie 2009,
Constana
Etapa naional a Olimpiadei, clasele V VI: 29 31 mai 2009
32
Probleme rezolvate din RMCS 24
Clasa a IV-a
IV.107 Numerele naturale a,b,c verific egalitile :
2 8, 3 4 18. a b c a b c + + = + + =
i)S se calculeze 2 3 ; a b c + +
ii) Dac 1 c = , exist un numr natural d astfel nct 2 10 a b d + = ?
Prof. Lucian Dragomir, Oelu-Rou
Rspuns: i) 2 3 13; a b c + + = ii) 1. d =
IV.108 Dac a este sfertul numrului 36, b este triplul numrului 10, iar
c este jumtatea numrului 18, s se arate c exist numerele x, y, z pentru
care: i) 3 a x b + = ;
ii) 2 c y b + = ;
iii) x z y + = . Inv. Elisaveta Vldu, Reia
Rspuns: 3, 12, 9. x y z = = =
IV.109 Dou stilouri i trei creioane cost 11 lei, iar cinci stilouri i patru
creioane cost 24 lei.
a) Ct cost un creion i un stilou?
b) Dac un stilou cost ct cost ct patru creioane, ct cost apte
creioane i dou stilouri?
Prof. Lucian Dragomir, Oelu-Rou
Rspuns: a) 5 lei ; b) 15 lei.
IV.110 Suma a cinci numere naturale consecutive este 2000. Calculai
jumtatea celui mai mic dintre cele cinci numere.
Inv. Elisaveta Vldu, Reia
Rspuns: 199.
IV.111 Liviu i Mihai au mpreun 24 de creioane. Jumtate din numrul
creioanelor lui Liviu este exact ct un sfert din numrul creioanelor lui
Mihai.Cte creioane are fiecare dintre cei doi?
Inst. Lidia Todor, Caransebe
Rspuns: 8 creioane are Liviu i 16 creioane are Mihai.
IV.112 Bogdan i alege un numr, l nmulete cu 2, apoi numrul
obinut l nmulete cu 3, iar noul numr obinut l nmulete cu 4.
Adunnd numrul 4 la ultimul numr la care a ajuns, Bogdan a obinut
numrul 100. Oana i alege i ea un numr, l nmulete cu 5 i, la
numrul obinut, adun numrul 5; Oana ajunge astfel tot la numrul 100.
Care dintre cei doi frai i-a ales un numr mai mare?
Prof. Marius andru, Reia
Rspuns: Oana(a ales numrul 19, pe cnd Bogdan a ales 4).
w
w
.
n
e
u
t
r
i
n
o
.
r
o
33
IV.113 ntr-un copac se odihnesc 11 psri, ciori i rndunele la un loc.
Un vntor alung printr-un foc de arm dou ciori i dou rndunele.
Dup dou focuri de arm, n copac rmn 3 ciori. Putei gsi cte
rndunele au fost la nceput n copac? (enun corectat)
Prof. Delia i Adrian Dragomir, Caransebe
Rspuns: 4 rndunele.
IV.114 Ioana are n curte dou zambile roii, dou lalele galbene, dou
lalele roii i dou zambile galbene. n cte feluri poate alege Ioana flori
din grdin pentru a alctui un buchet format din trei flori de aceeai
culoare?
Prof. Delia i Adrian Dragomir, Caransebe
Rspuns: 4 feluri (2 zambile, o lalea galbene, 2 zambile, o lalea roii,
2 lalele, o zambil galbene, 2 lalele, o zambil roii). (Florile nu sunt
numerotate, deci nu putem face distincie ntre dou zambile galbene, de
exemplu).

Clasa a V-a
V.107 Pentru fiecare nNse noteaz 5 2, 5 3 a n b n
n n
= + = + .
a) S se arate c pentru orice nN, exist k N astfel nct
5 k a b
n n
= + ;
b) S se arate c nu exist , , i j k Nastfel nct b a a
i j k
= + ;
c) S se arate c exist cel puin trei numere naturale n pentru care
a b
n n
+ este ptrat perfect ;
d) S se arate c nu exist nN pentru care cel puin unul dintre
numerele a
n
i b
n
este ptrat perfect ;
e) Exist nN pentru care a b
n n
este ptrat perfect ?
Prof. Adriana Dragomir, Oelu-Rou
Soluie: a) 2 1; k n = + b) presupunem prin absurd c b a a
i j k
= + i
ajungem la 5 3 5( ) 4 5 4 k i j q + = + + = + , imposibil ; c) de exemplu,
{ } 2, 22, 62 n ; d) ultima cifr poate fi 2 sau 7 , respectiv 3 sau 8 ; e) nu
exist, deoarece, de exemplu, ( ) ( )
2 2
5 2 5 3 n a b n
n n
+ < < + .
V.108 Patru prieteni practic fiecare cte un alt sport i poart pe tricouri
numere diferite.Se tie c :
a) Bogdan nu joac baschet i nu are pe tricou numrul 7;
b) Laureniu joac fotbal;
34
c) Daniel are pe tricou numrul 5;
d) Mihai nu are numrul 6 i nu joac baschet ;
e) Cel care joac handbal are numrul 7;
f) Cel care are numrul 10 joac fotbal.
Putei gsi ce sport practic Bogdan, ce numr are Bogdan pe tricou
i ce sport practic Daniel?
Prof. Marius andru, Reia
Soluie: Laureniu joac fotbal i are numrul 10; Bogdan nu are numrul
7, deci nu joac handbal; deoarece nu joac nici baschet i nici
fotbal(Laureniu joac fotbal), Bogdan practic cel de-al patrulea sport
(cum nu se indic n ipoteze nimic despre acesta, poate fi oricare altul:
nataie, tenis, tir, etc. Oricum, nu putem preciza ce sport practic
Bogdan). Pe de alt parte, Bogdan nu are numrul 5( l are Daniel), nici 7,
nici 10, deci are numrul 6. Acum, ajungem la concluzia c Daniel
practic baschetul. Asta a fost. (De ultim moment, Bogdan ne-a anunat
c merge mari i joi la not i n fiecare dup-amiaz, cu tatl su, la tenis
de cmp. Mulumim mult i i dorim succese majore n sport i la
matematic.)
V.109 S se gseasc numrul perechilor ( ) , a b de numere naturale
pentru care ( )
2
36 a a b + = . Prof. Loreta Ciulu, Reia
Rspuns: 3 perechi: ( ) 1, 35 , (2, 7), (3,1).
V.110 Pentru fiecare numr natural n , se noteaz (5 4)( 7)
n
a n n = + + .
a) S se arate c pentru orice nN, a
n
este numr par;
b) S se determine cte elemente are mulimea { } / 2008 M n a
n
= < N
Prof. Loreta Ciulu, Reia
Soluie: a) Metoda clasic: se consider, pe rnd, cazurile n par, n impar;
b) testri rapide conduc la ( ) 17. card M =
V.111 S se studieze dac numrul
2006 2007 2008
2005 2006 2007 A = + +
este ptrat perfect.
Prof.Marius andru, Reia
Soluie: Ultimele cifre ale celor 3 termeni sunt 5, 6, respectiv 1, deci
ultima cifr a numrului A este 2, aadar A nu poate fi ptrat perfect.
V.112 Un grup de patru numere naturale (nu neaprat distincte) se
numete grup frumos dac suma a dou dintre numere este egal cu suma
celorlalte dou; de exemplu, grupurile ( ) 1,1, 3, 3 i ( ) 2, 4, 5, 7 sunt
frumoase.
w
w
.
n
e
u
t
r
i
n
o
.
r
o
35
a) S se determine x N pentru care grupul ( ) , 3, 3 , 7 x x este frumos;
b) Dac { } 1, 2, 3, ..., 2008 x , s se determine cte grupuri frumoase de
forma ( ) , 1, 6, 7 x x + exist ?
Prof. Lucian Dragomir, Oelu-Rou
Rspuns: a) 2; x = b) 2008 grupuri.
V.113 ntr-o familie de 4 persoane suma vrstelor acestora este de 97 de
ani. Biatul s-a nscut cnd tatl avea 23 de ani, iar fata s-a nscut cnd
mama avea 22 de ani i fratele su 4 ani. Putei gsi ce vrst are fiecare?
Prof. Mariana Drghici, Reia
Rspuns: 11 ani fata , 15 ani biatul, 33 ani mama, 38 ani tata.
V.114 S se gseasc cele mai mici trei numere naturale a,b,c consecutive
a cror sum este un numr care are ultimele patru cifre 2,0,0,8.
Prof. Adriana Dragomir, Oelu-Rou
Soluie: 1, , 1 3 a n b n c n a b c n = = = + + + = , aadar numrul
... 2008
1 2
a b c a a a
m
+ + = este divizibil cu 3; deoarece
2 0 0 8 10 3 3 1 + + + = = + , suma ...
1 2
a a a
m
+ + + trebuie s fie de forma
3 2 k + . Cele mai mici numere se obin pentru
... 2 22008 7335, 7336, 7337
1 2
a a a a b c a b c
m
+ + + = + + = = = =
Observaie : puteam ncerca i aa:
( 1) ( 2) 2008 3 2005 a a a a + + + + = = , imposibil n N;
( 1) ( 2) 12008 3 12005 a a a a + + + + = = , la fel i, n sfrit,
( 1) ( 2) 22008 7335 a a a a + + + + = = , etc.

Clasa a VI-a
VI.107 Se consider numerele
1 1 1 1
...
4 5 6 2008
A = + + + + i
3 4 5 2007
...
4 5 6 2008
B = + + + + .
a) S se arate c A B + este numr natural;
b) S se arate c
2005
2005
2
B A < < .
Prof. Delia i Adrian Dragomir, Caransebe
Soluie: a)
1 3 1 4 1 2007
... 2005 ;
4 4 5 5 2008 2008
A B + = + + + + + + =



N
36
b) 2005 2 0 B A B A A B A < < + > , adevrat; pentru prima
inegalitate, s observm c
2 3 4 2006 1 1 1 2005
... ...
4 5 6 2008 2 2 2 2
B A = + + + + > + + + = .
VI.108 Numerele naturale , , 4 a b sunt direct proporionale cu numerele
2, 3, respectiv c . Poate fi numrul a b c + + ptrat perfect ?
Prof. Marius andru, Reia
Rspuns : Da, numai pentru 2, 3, 4. a b c = = = (Artai c aceasta este
singura posibilitate!).
VI.109 n triunghiul ABC mediatoarea laturii (BC) intersecteaz latura
(AC) n punctul D. Dac ( ) 3 ( ) m ABC m ACB = X X i ( ) ( ) AB BD ,
putei determina msurile unghiurilor triunghiului ABC?
Istodor Cosmin, elev, Oelu-Rou
Rspuns:
0 0 0
90 ,30 , 60 .
VI.110 Se consider o mulime M care satisface urmtoarele proprieti;
a) dac x M ,atunci
3 1
;
2
x
M
+

b) dac
4 1
3
x
M
+
,atunci x M .
S se arate c:
i) dac 1 M ,atunci
7
2
M ; ii) dac 3 M ,atunci
23
4
M .
Prof. Heidi Feil, Oelu-Rou
Soluie : i)
) )
7
1 2 ;
2
a a
M M M ii)
) )
3 5 8
a a
M M M i deci
)
23
4 1
23
4
8 .
3 4
b
M
+
=
VI.111 S se determine cifra x tiind c numrul

1 1 1
0, ( ) 0, 0( ) x x x
+ + este numr natural.
Gazeta Matematic 2002
Rspuns: { } 1, 2, 4,5 . x
VI.112 ntr-un triunghi lungimile laturilor sunt exprimate prin numere
naturale pare. Dac lungimea unei laturi a triunghiului este egal cu 2,
artai c triunghiul este isoscel.
w
w
.
n
e
u
t
r
i
n
o
.
r
o
37
Concurs Iai
Soluie: Dac lungimile laturilor sunt 2, 2 , 2 p q , putem presupune
, , p q p q

> N i, din 2 2 2 p q < , deducem 1 . p q p q < =
VI.113 S se determine numerele naturale nenule a, b, c pentru care

1 1 1 a b c
b c a
+ + +
= = N.
Prof. Delia i Adrian Dragomir, Caransebe
Soluie:
1 1 1 3 a b c
b c a
a b c
a b c
+ + +
= =
+ + +
=
+ +
N , de unde
3
1
a b c
+
+ +
Ni deci ( ) { } 1, 3 a b c + + ; cum numerele sunt naturale
nenule, ajungem la 1. a b c = = =
VI.114 Segmentele (AB) i (CD) sunt incluse n dreapta d. Se tie c
7 AB cm = i 5 CD cm = . Indicai un mod de a delimita pe dreapta d,
numai cu ajutorul compasului, un segment (BE) cu lungimea de 1cm.
Prof. D.M.Btineu-Giurgiu, Mircea Fianu, Bucureti
Soluie: Lum n deschiztura compasului [ ] CD i punem vrful n A;
obinem astfel un punct F ntre A i B cu 5 2 AF FB = = .Cu aceeai
deschidere, aezm vrful n F i obinem punctul G cu 5 3 FG BG = = ;
lum n compas acum 2 FB = i aezm vrful n G i vom ajunge la
2 1 FB GE BE = = = .

Clasa a VII-a
VII.107 S se arate c dac dou numere reale mai mari dect 1 au
produsul egal cu 2, atunci suma lor este mai mic dect 3.
Prof. Nistor Budescu, Dalboe
Soluie: Dac 2 x ,deoarece 1 y > ,am avea 2 xy > , contradicie cu
ipoteza. Aadar 2 x < i,la fel, 2. y < Considerm
1 , 1 , 0 1, 0 1. x a y b a b = + = + < < < < Din 2 xy = deducem
1 a b ab + + = i, deoarece 0 1 ab < < , ajungem la
1 1 1 1 3. a b ab x y a b + = < + = + + + <
Metoda 2 (Adriana Dragomir) :
2
, 1, 2 a b a b b
a
> = = i avem de artat
c
2
3 a
a
+ < care este echivalent cu ( ) ( ) 1 2 0 a a < ...
38
VII.108 Bogdan scrie pe tabl dou numere, iar Oana le terge i scrie
produsul i modulul diferenei lor. Pe tabl sunt scrise i acum ns
numerele scrise de Bogdan.
Putei gsi ce numere a scris Bogdan pe tabl ?
Prof. Marius andru, Reia
Rspuns: Dac a i b sunt numerele scrise de Bogdan pe tabl, atunci
( ) ( ) { }
, 1, 2 , (2,1), ( , 0), (0, ) / , 0 . a b a b a b
VII.109 S se gseasc mulimile A i B care au fiecare cte 3 elemente
tiind c satisfac urmtoarele proprieti:
a) 4 ; A B
b)
2
x A x B ;
c) suma elementelor mulimii B este triplul sumei elementelor mulimii A.
Prof. Marius andru, Reia
Soluie: De remarcat c, n forma publicat(eroare de tehnoredactare din
cauza vitezei sau a oboselii, ne cerem scuze), condiia b) era
2
x A x A , situaie n care am fi avut
2 4
4 4 4 A A A ,
deci A nu ar fi avut doar 3 elemente (toi cei care au observat acest fapt au
primit, pe lng punctajul maxim, un bonus de 5 puncte). Cu enunul n
forma actual (problem rezolvat i aa de ctre unii elevi, care au primit
i ei punctajul corespunztor), avem : { }
{ }
2
2, 4, , 4,16, A b B b = = i ,
folosind c), ajungem la { } { } 1, 2, 4 , 1, 4,16 . A B = =
VII.110 Fie ABCD un dreptunghi i ( ) M CD cu 2 2 . MD MC BC = = S
se determine msura unghiului ascuit determinat de dreptele BD i AM.
Olimpiad Iai
Soluie: O problem frumoas(prerea noastr). Fr a diminua din
generalitatea problemei, pentru a uura unele calcule, putem considera
1 BC = i astfel avem 1, 2, 3. MC MD DC = = = Dac E este simetricul lui
C fa de B , teorema lui Pitagora conduce la 5, AM ME = =
10 AE DB = = (AEBD este paralelogram !), deci AME este triunghi
dreptunghic isoscel; cum // BD AE , deducem c msura cutat este
( )
0
45 . m MAE = X
VII.111 Se consider numerele naturale nenule a,b,c,d care
satisfac:
1
1
a b ab
c d cd
+
= =
+
. S se calculeze
a d
b c
+
+
.
Concurs Rusia
w
w
.
n
e
u
t
r
i
n
o
.
r
o
39
Soluie: Considerm valoarea comun a rapoartelor ca fiind k i ajungem
imediat la ( ) 1 ( 1) 0 1 kcd k k = = , aadar , a c b d = = 1.
a d
b c
+
+
=
VII.112 Pentru un examen, comisia a pregtit 8 probleme. Fiecare student
primete spre rezolvare cte 3 probleme, oricare doi primind cel mult o
problem comun. Care este numrul maxim de studeni care pot
participa n aceste condiii la examen ?
Concurs Baltic
Soluie: Numrul maxim cutat este 8. Presupunem c exist o problem
dat la cel puin 4 studeni i astfel avem c acetia mai trebuie s
primeasc 2 4 8 = probleme diferite, imposibil. Aadar, orice problem a
fost dat la cel mult 3 studeni. Deoarece avem 8 probleme, numrul
maxim de studeni este
8 3
8.
3

= S artm acum c aceast situaie este i


posibil. Numerotm problemele cu 1,2,3,4,5,6,7,8 i avem, de exemplu,
urmtoarea repartiie pentru cei 8 studeni: ( ) ( ) ( ) 1, 2,3 , 3, 4, 5 , 5, 6, 7 ,
( ) ( ) ( ) ( ) ( ) 7,8,1 , 2, 4, 7 , 4, 6,1 , 6,8, 3 , 8, 2,5 .
VII.113 S se determine numerele prime a,b,c care verific egalitatea
5 10 a b c a b c + + = .
Prof.Aurel Brsan,Braov
Soluie : Pentru nceput, s presupunem c c b a . Avem astfel
5 10 16 abc a a a a + + = , de unde 16 bc . Analizm subcazurile posibile
(numere prime), apoi nc 5 mari cazuri cu subcazurile posibile. Ajungem
astfel la: 5, 5, 2 a b c = = = sau 7, 3, 2 a b c = = = sau 3, 23, 2. a b c = = =
VII.114 Se consider triunghiul ABC n care P este mijlocul laturii
(BC).Fie ( ) ( ) , M AB N AC astfel nct // MN BC i { } . Q MP BN =
Perpendiculara din Q pe dreapta AC intersecteaz pe AC n R i paralela
prin B la AC n T. S se arate c :
a) // ; TP MR b)

MRQ PRQ .
Prof.Mircea Fianu, Virgil Nicula, Bucureti
Soluie: O problem nu chiar uoar, mai ales n partea a doua !.
a) // BT AC conduce la
QT QB
QTB QRN
QR QN
= (1) i // MN BC
conduce la
QP QB
QBP QNM
QM QN
= (2). Din (1) i (2) avem
40
QT QP
QR QM
= , de unde // MQR PQT MR PT . b) Notm
{ } TP AC S = i, din , // BP PC TB CS = , avem c TBSC este
paralelogram, deci P este mijlocul i al lui [ ] ST . Deducem c n
triunghiul dreptunghic TRS avem
1
2
RP TS TP = = , de unde
. TRP RTP X X Pe de alt parte, avem i RTP MRT X X (alterne
interne...), aadar . TRP MRT X X
Clasa a VIII-a
VIII.107 Graficele funciilor , : , ( ) , ( ) , 0 f g f x ax b g x cx d ac = + = + R R
sunt cele din desenul de mai jos, iar punctul de pe axa Oy este (0, 2). A


S se arate c dac triunghiul
ABC este dreptunghic n A i are
aria egal cu 4,atunci triunghiul
este isoscel.
Prof.Lucian Dragomir,Oelu-Rou
Soluie : (0) 2, (0) 2 2, 2 f g b d = = = = ; pe de alt parte avem
2 2
, 0 , , 0 , 0, 0 B C a c
a c
> <



. Cu teorema lui Pitagora avem
imediat
4 4
4 , 4
2 2
AB AC
a c
= + = + i
2
4 4 2 2
4 4
2 2
c a
a c
+ + + = +



;
deducem imediat 1 ac = .Deasemenea avem i
2 2
1 1
( ) 4 1 1 4
2
AB AC
ABC
a c

= = + + =



A . Folosind
1 ac = ajungem la 1, 1 a c = = i concluzia este imediat.

VIII.108 S se determine numerele prime p,q pentru care
2 2
2 1 p q = .
* * *
Soluie: Evident, p trebuie s fie numr impar. Se observ imediat soluia
3, 2 p q = = .Dac ambele numere sunt impare, adic
2 1, 2 1, , p k q m k m = + = + N, prin nlocuire ajungem imediat la
( )
2 2
2 2 2 1 k k m m + = , absurd.
w
w
.
n
e
u
t
r
i
n
o
.
r
o
41
VIII.109 S se calculeze aria maxim a unui patrulater cu lungimile
laturilor 1, 4, 7 i 8.
* * *
Soluie: Se poate presupune c laturile cu lungimile 1 i 8 sunt alturate
(n caz contrar, tiem patrulaterul dup o diagonal i rsturnm unul
dintre triunghiuri). Aria patrulaterului este astfel cel mult egal cu
8 7
1 4 18
2 2
+ = . Deoarece
2 2 2 2
1 8 4 7 65 + = + = , patrulaterul cu aria
maxim de 18 poate fi chiar obinut(din dou triunghiuri dreptunghice cu
ipotenuzele egale cu 65 ). O problem frumoas, incitant, nu foarte
uoar. Metoda 2 (prezentat de elevul Marian Marta): Folosim formula
lui Arhimede conform creia aria patrulaterului satisface
2 2
( )( )( )( ) cos
2
A C
S p a p b p c p d abcd
+
= ; aceasta este
maxim dac cos 0
2
A C +
= , adic
0
180 A C + = , adic patrulaterul este
inscriptibil; urmeaz simple nlocuiri.
VIII.110 S se determine numerele ntregi a i b care satisfac:

1 1 4
3 a b
+ = i
2 2 a b
b a
+
= .
Prof. Ovidiu Bdescu,Reia
Soluie:
2 2 2 2
1
a b a b
b a b a
+ + +
= = =
+
conduce la 2 b a = + . De aici se
ajunge imediat la 1, 3. a b = =
VIII.111 S se determine numerele ntregi m pentru care

3 2
2
3 2
m
m m
+

+ +
Z.
Prof. Marius andru, Reia
Rspuns: 0. m =
VIII.112 Un elev are 10 bile numerotate cu numerele 1,2,3,...,9,10. El
trebuie s le pun n trei urne identice astfel nct n nici o urn s nu fie
dou bile numerotate cu numere consecutive. n cte cte moduri se poate
face acest lucru? Shortlist, ONM, 2002
Soluie: Bila cu numrul 1 poate fi pus n oricare urn, deci avem 3
posibiliti. Bila cu numrul 2 poate fi pus n dou urne(cele care nu
conin bila 1), la fel fiecare dintre bilele urmtoare. Avem n total, cu
principiul produsului,
9
3 2 modaliti posibile.
42
VIII.113 a) S se arate c pentru orice , , x y z Reste adevrat
inegalitatea ( ) ( )
2
3 x y z xy yz zx + + + + ;
b) Dac a,b,c sunt numere reale pozitive cu 1 abc = , s se arate c:

3 6
1
a b c ab bc ca
+
+ + + +
.
Prof. Mircea Lascu, Vasile Crtoaje
Soluie: b) Notm
1 1 1
, , x y z
a b c
= = = (evident, 1 xyz = ) i inegalitatea
propus devine
3 6
1
xy yz zx x y z
+
+ + + +
; folosind a) deducem c
( )
3 9
1 1
2
xy yz zx
x y z
+ +
+ +
+ +
i astfel este suficient s demonstrm
c
2
9 6
1
( )
x y z
x y z
+
+ +
+ +
. Aceasta este ns echivalent cu
2
3
1 0
x y z



+ +

, care e evident adevrat.
VIII.114 Se consider dreptunghiurile ABCD i AEFG astfel nct
punctele B,E,D,G s fie coliniare(n aceast ordine). Dac T este punctul
de intersecie al dreptelor BC i FG, iar H punctul de intersecie al
dreptelor DC i FE, s se arate c punctele A,H i T sunt coliniare.
Prof. Mircea Fianu, Bucureti
Soluie : Fie { } { } , CD FG M BC EF N = = .Observm c
patrulaterele DHEA i FHCT sunt insriptibile (unghiuri opuse drepte) i
astfel ajungem la
0
180 FTC FHC DAE = = X X X i DAH DEH = X X .
Din
0 0
90 90 DMG FTC DAE DAG = = = X X X X avem c i ADGM
este patrulater inscriptibil. Mai departe obinem: DAM FGE = X X i
0
90 MAH DAM DAH FGE DEH = + = + = X X X X X . Analog,
0
90 NAH = X , adic punctele M,A,N sunt coliniare. Deoarece n
triunghiul TNM punctul H este ortocentru, deducem c A,H,T sunt
coliniare (situate pe nlimea AT).



w
w
.
n
e
u
t
r
i
n
o
.
r
o
43
Clasa a IX-a
IX.107 S se arate c :
[ ]
[ ] 1 1 5
2 , 0
1 1 2
x x
x
x x
+ +
+
+ +
, unde
[ ] a reprezint partea ntreag a numrului real a.
Prof. Pavel Rncu, Dalboe
Soluie: Inegalitatea din stnga e imediat folosind inegalitatea
mediilor: 2 2
a b a b
b a b a
+ = . Pentru cealalt notm
[ ] { } [ ] , , 1 1 x n x k x n k x n = = = + + = + i inegalitatea propus devine
1 1 5
1 1 2
n k n
n n k
+ + +
+ <
+ + +
. Calcule nu foarte complicate conduc la
inegalitatea echivalent ( 2 1)( 1) 0 n k n k + + + > , care e evident
adevrat (!).
IX.108 S se rezolve ecuaia:
5 9
4 9 9
x
x x
= +
+ +
.
Prof. Alfred Eckstein,Viorel Tudoran, Arad
Soluie:
36
5 4
9 9
x
x x
= +
+ +
sau
( 9) 36 5 9
5 4 4
9 9 9 9
x x x x
x x x x
+ +
= =
+ + + +
. Imediat ajungem acum la
( )
2
4 9 0 x x + = ,de unde 3. x =
IX.109 Se consider numerele reale , , , 0 a b c a ,pentru care
2
ac b > .S
se arate c dac 2 a c b + > , atunci 4 4 . a c b + >
Prof. Lucian Dragomir, Oelu-Rou
Soluie: Se consider
2
: , ( ) 2 f f x ax bx c = + R R .Conform ipotezei
avem 0 < , deci f are semn constant pe R.Cum 2 a c b + > nseamn de
fapt (1) 0 f > ,deducem ( ) 0, f x x > R i 0 a >
(2) 4 4 0 f a b c = + > i concluzia e imediat.



44
IX.110 S se arate c dac ( ) , 0, 2 x y i 1 x y = , atunci

2 2
2
2 2
x y
y y x x
+

.
Prof. Ovidiu Bdescu, Reia
Soluie: ( )
2 1
, 0, 2 (2 ) 1 1
2 (2 )
x x
x y x x
x x
+
=

i deci
(2 )
x
x
x x

.Analog avem i
(2 )
y
y
y y

. Prin nsumare i folosind


2 2 x y xy + = se ajunge la inegalitatea propus.

IX.111 S se determine aR pentru care exist , x y Z astfel nct

2
2 2
2
x xy a
y xy a
=
+ =

.
Prof. Lucian Dragomir, Oelu-Rou
Soluie: Dac 0 0 a x y = = = Z; dac 0 a , nmulim prima
ecuaie cu a i adunm cele dou ecuaii, apoi mprim ecuaia obinut
cu
2
0 x . S mai remarcm c , x y Z conduce la aZ i, cu notaia
y
t
x
= avem
2
2 ( 1) 0 t a t a + + = . Aceast ecuaie trebuie deci s
aib rdcini raionale, deci exist k Z astfel nct
2 2
( 5) 24 a k = + = , de unde ( 5 )( 5 ) 24 a k a k + + + = . Analizm
cazurile posibile i ajungem n final la { } 12, 0, 2 . a
IX.112 Se noteaz cu O i H centrul cercului circumscris, respectiv
ortocentrul triunghiului ABC. S se arate c dac HA OA = i
HB OB = , atunci . HC OC =
Prof. Lucian Dragomir, Oelu-Rou
Soluie: 2 cos
3
HA OA R A R A

= = = ; analog
3
B

= i finalizarea
trebuie s v fie la ndemn.(Demonstrai c cos AH R A = )

IX.113 S se determine funciile : g N N tiind c:
a) ( 1) ( ) ( 1) 1 , ; x g x x g x x + + = N
w
w
.
n
e
u
t
r
i
n
o
.
r
o
45
b) (4), (12), (24) g g g sunt ptrate perfecte.
Prof. Lucian Dragomir, Ovidiu Bdescu
Soluie: Se obine imediat inductiv c ( ) 1, g x ax a = + N. Problema este
n continuare, cum folosim b)? Din
2 2 2
4 1 ,12 1 , 24 1 a k a p a q + = + = + =
ajungem la condiia ca
2
3 2 k i
2
6 5 k s fie simultan ptrate perfecte.
Se obin imediat 1, 3 0, 2 k k a a = = = = (rularea pe calculator a unui
program pentru { } 1, 2,..., 20000 k nu a condus la alte valori; n acest
moment, redacia nu dispune de o soluie complet i suficient de uoar a
problemei, care s evite rezolvarea unor ecuaii de tip Pell). Aadar, avem
deocamdat dou soluii :
1 2
( ) 1, ( ) 2 1 g x g x x = = + ...... Ateptm.
IX.114 Graficul funciei
2
: , ( ) , , f f x x ax b a b = + + R R R , este
cel din desenul de mai jos:
a) Dac (0,1) A i B este punctul de
tangen cu Ox, putei determina a i b ?
b) S se arate c exist C i D puncte
distincte pe graficul funciei date astfel
nct ariile triunghiurilor ABC i ADC
reprezint un acelai numr ntreg.


Prof. Lucian Dragomir, Oelu-Rou
Soluie: a) Din (0) 1 f = deducem 1 b = . Vrful parabolei este
, 0
2
a
B



;din 0
2
a
f

=


, ajungem la 2 a = .Cum ns 0
B
x > , avem
doar 2 a = i deci ( )
2
( ) 1 ; f x x = b) Cel mai simplu ar fi s lum
(2,1)
f
C G i pentru care [ ] 1 ABC = Z A ; acum, s observm c
pentru orice punct M situat pe dreapta orizontal de ecuaie 2 y = avem
[ ] 1 AMC = Z A i astfel punctul D este intersecia acestei drepte cu
parabola noastr, adic (1 2, 2) D + .
46
Clasa a X-a
X.107 S se rezolve n mulimea numerelor reale ecuaia:

6
1 1 1 1
3
4 27 3 2
x
x x
+ = +





.
Prof. Alfred Eckstein, Viorel Tudoran, Arad
Soluie : Notm
1
,
3
2
a =
1
, 6
3
b x t = = i astfel ecuaia se poate scrie
( )
t
t t
a b a b + = + ; considerm : , ( )
t t
a b
f f t
a b a b
= +
+ +



R R
care este strict cresctoare, deci injectiv. Aadar ( ) (1) 1 f t f = = conduce
la
1
1, .
6
t x = =
X.108 S se arate c pentru orice n

N , numrul
( ) 3 !
3
( !)
n
n
este natural.
* * *
Soluie:
( ) ( )
( )
( ) 3 ! 3 ! 2 !
2
3 2
3
( !) 2 ! ( !)( !)
( !)
.
n n n
n n
C C
n n
n n n n
n
= = N
X.109 S se determine numrul funciilor { } { } : 1, 2, 3, 4, 5, 6, 7 0,1 f
pentru care (1) (2) ... (7) 3 f f f + + + = .
Elev Istodor Cosmin, Oelu-Rou
Soluie: Exact 3 dintre imaginile elementelor din domeniu trebuie s fie
egale cu 1, restul cu 0 ; aceasta se poate alege n
3
7
35 C = de feluri.
X.110 S se arate c exist un singur numr ntreg x pentru care

2
2
2
log
1
x
x x
x
+ =
+
.
Prof. Ovidiu Bdescu, Reia
Soluie: Din cauza unei erori de tehnoredactare, egalitatea din enun nu
este verificat de niciun numr ntreg ( 0, 1 x x x > Z i astfel
membrul stng este cel puin egal cu 1, pe cnd cel drept este cel mult
egal cu 1/ 2 ). Toi elevii care au sesizat aceast eroare primesc, evident,
punctaj maxim.V propunem nlocuirea ecuaiei cu:
2
2
log
2
1
x x
x
x
+ =
+
.
w
w
.
n
e
u
t
r
i
n
o
.
r
o
47
X.111S se gseasc n cte feluri poate fi ales un numr impar de obiecte
avnd la dispoziie n obiecte. * * *
Soluie: ntre mulimile cu numr impar de obiecte i cele cu numr par
de obiecte exist o coresponden bijectiv, deci numrul cutat este exact
jumtate din numrul total de submulimi ale unei mulimi cu n elemente
(obiecte), adic
1
2 .
n

X.112 Se consider 3 1 n + obiecte, dintre care n sunt identice, iar restul
diferite.S se determine n cte feluri se pot alege n obiecte dintre cele
3 1 n + considerate. * * *
Soluie: Se pot alege cele n obiecte n
1 0
2 1 2 1 2 1
...
n n
n n n
C C C

+ + +
+ + + moduri;
adunnd acestuia acelai numr, scris astfel :
1 2 2 1
2 1 2 1 2 1
...
n n n
n n n
C C C
+ + +
+ + +
+ + + .obinem
2 1
2
n+
i astfel numrul cutat de
posibiliti este
2
2 .
n

X.113 S se calculeze
3
1
n
k
n n
k
S k C
=
=

. * * *
Soluie(Lucian Dragomir): Vom parcurge cteva etape. Pentru nceput,
1
1
0
2
n
k n
n
k
T k C n

=
= =

( rezultat destul de cunoscut, uor de gsit). Acum,


avem
2 2 2
2 1 1 2
0 0 2
( 1)
( ) ( 1)
( 1)
n n n
k k k
n n n
k k k
n n
T k C T k k C T k k C
k k

= = =

= = + = + =



2 2 2
1 2 1
2
( 1) ( 1) 2 ( 1) 2
n
k n n
n
k
T n n C T n n n n

=
= + = + = +

. Analog,
3 2 2 2
3 2 2 2
0 2
( 1)
( ) ( 1)
( 1)
n n
k k
n n n
k k
n n
T S T k k C T k k C
k k

= =

= = + = + =



2 3 1 2 3
2 2 2
2
( 1) ( 2 2) ( 1)( 2)2 ( 1)2 ( 3)2 .
n
k n n n
n
k
T n n k C T n n n n n n n

=
+ + = + + = +

X.114 S se arate c n orice triunghi de laturi a,b,c este adevrat


inegalitatea
( ) ( ) ( )
2 2 2 2 2 2 2 2 2
3 a b c a b c a b c a b c abc + + + + + .
* * *
Soluie: Cu teorema cosinusului, inegalitatea propus devine
3
cos
2
A

; aceasta se gsete n orice carte serioas tematic...


48
Clasa a XI-a
XI.107 a) S se arate c exist cel puin dou funcii derivabile
: f R R pentru care
2
'( ) 1, f x x x x = + + R ;
b) S se arate c exist o singur funcie derivabil : g R R
pentru care (0) 0 g = i '( ) ( ).
x
g x e g x = +
Prof. Nicolae Stniloiu, Boca
Soluie: (Lucian Dragomir) a) Orice funcie : f R Rde forma
3 2
1 1
( )
3 2
f x x x x k = + + + , unde k R, satisface enunul!;
b) Presupunem, prin absurd, c exist dou funcii g, h care satisfac
condiiile din enun i obinem imediat c
/ /
g h g h = , de unde
( ) ( )
x
g x h x e k = + ; cum (0) (0) g h = , deducem 1 k = i deci
( ) ( ) 1
x
g x h x e = , de unde
/ /
( ) ( )
x
g x h x e = i imediat, nlocuind n
egalitatea din enun, avem ( ) ( )
x
g x h x e = , contradicie, aadar exist cel
mult o funcie cu proprietatea din enun.Rmne de artat c exist o
astfel de funcie!. Observm c ( )
x
g x xe = satisface enunul.
XI.108 S se demonstreze c sin cos 1 x x x + , 0,
2
x





.
* * *
Soluie: Problem clasic.Se consider : 0,
2
f




R,
( ) sin cos f x x x x = + , care este derivabil i se arat imediat c este
cresctoare, deci ( ) ( ) 0 , f x f 0,
2
x




.(observaie: n forma
publicat, inegalitatea era fals, dup cum a observat elevul Harald
Nezbeda, de exemplu).
XI.109 S se arate c dac numerele reale a,b,c, 0 c , satisfac relaiile
2 3 3, 3 4 4, 2 1 a b c a b c a b c + + = + + = + = , atunci 0. a b c +
Prof. Lucian Dragomir, Oelu-Rou
Soluie: Primele trei egaliti conduc la un sistem de ecuaii, compatibil
nedeterminat; se obine imediat 1 , 1 , a c b c = = de unde
0. a b c c + =
w
w
.
n
e
u
t
r
i
n
o
.
r
o
49
XI.110 S se demonstreze c
2
e
e e
e

< < .
Prof. Nicolae Stniloiu, Boca
Soluie: Se aplic teorema lui Lagrange funciei [ ] : , , ( ) ln . f e f x x x = R
XI.111 Pentru orice funcii derivabile , : f g R Rse noteaz
{ } / ( ) ( ) A x f x g x = = R . S se arate c dac { } 0 A = ,atunci exist
( ) 0,1 c pentru care
/ /
( ) ( ) 1
( ) ( ) 1
f c g c
f c g c c

=

.
Prof.Lucian Dragomir,Oelu-Rou
Soluie: Se aplic teorema lui Rolle pe [ ] 0,1 funciei derivabile
[ ] : 0,1 , ( ) ( ( ) ( )) (1 ) h h x f x g x x = R .
XI.112 Se consider matricea ( )
1 2
2
2 3
A

=




Z M .
a)S se arate c pentru orice ( )
2,1
B Z M , exist o unic matrice
( )
2,1
X Z M astfel nct A X B = ;
b) S se arate c exist , m nZ astfel nct
3
2
A mA nI = + ;
c) S se determine numrul perechilor ( , ) x y de numere
ntregi pentru care
2
2 3
3 5 7
x y
x y
x y


Prof. Lucian Dragomir, Oelu-Rou
Soluie: a) Aeste inversabil, deci
1
X A B

= ; b) calcul direct sau relaia


lui Hamilton-Cayley conduc la 3, 2; m n = = c) Primele dou inegaliti
conduc la 2 , 2 3 x y a x y b = = N N, cu 2 3 , 2 x b a y b a = = ;
ultima inegalitate conduce astfel la 7 b a + i obinem imediat
8 7 ... 1 36 + + + = perechi ( ) , a b N N, deci i 36 perechi care satisfac
condiiile din enun.
50
XI.113 Graficul funciei
: , f R R
3
( ) , 0 f x ax bx c a = + +
este cel alturat. S se arate c
0 abc < .
Prof. Lucian Dragomir, Oelu-Rou
Soluie: lim ( ) 0
x
f x a

= + > ; (0) 0 f c = > i, deoarece f are dou


puncte de extrem distincte, ecuaia asociat derivatei,
2
3 0 ax b + = , are
dou rdcini reale distincte, de unde 0. b < Concluzia e imediat.
XI.114 Se consider funcia ( ) R , : f 0 , ( )
2
1
x
x f = .
a) S se arate c f este strict descresctoare pe ( ) , 0 ;
b) S se arate c :
144
52 1 1
144
25
2 2
< + <
e
.
Prof. Lucian Dragomir, Oelu-Rou
Soluie : a) banal ; b) se adun inegalitile (3) ( ) (2) f f e f < < i
(4) ( ) (3). f f f < <
Clasa a XII-a (a XI-a)
XII.107 Pentru orice numere reale x i y se noteaz x y x y xy = + + .
a) S se arate c exist aZ astfel nct
( ) ( 1)( 1)( 1) , , , x y z x y z a x y z = + + + + R ;
b) S se calculeze
1 1 1
1 ...
2 3 2008
;
c) S se determine toate numerele reale b pentru care: , x y b x y b
* * *
Soluie: a) 1; a = b) deoarece ( 1)( 1) 1 x y x y = + + ,artm prin
inducie c ( ) ( ) ( )
1 2 1 2
* ...* 1 1 ... 1 1
n n
x x x x x x = + + + i astfel
ajungem la
1 1 1
1 ...
2 3 2008
2008 = ; c) 1. b =




w
w
.
n
e
u
t
r
i
n
o
.
r
o
51
XII.109 Pentru orice ( ) , 5, x y se noteaz 5 5 30 x y xy x y = + .
a) S se arate c ( ) 5, x y ;
b) S se arate c pentru orice ( ) 5, a , exist ( ) 5, b astfel
c) nct 6 a b = ;
d) S se determine , , x y z pentru care
x y z
y z x
z x y
=
=
=


* * *
Soluie: Problem de bacalaureat. c) soluie unic, tripletul ( ) 6, 6, 6 .
XII.110 Se consider funcia ( ) : 0, , ( ) ln ,
n
n
f f x x x n
n

= + R N
S se demonstreze c dac
n
x este soluia ecuaiei ( ) 0
n
f x = , atunci irul
( )
1
n
n
x

este convergent. S se determine limita irului ( )
1
n
n
x

.
* * *
Soluie: Considerm ( ) ( ) : 0, , ln
n
f f x x x = + R , care este continu
i strict cresctoare. Cum ( )
0
0
lim
x
x
f x

>
= , avem c exist 0 > cu
( ) 0; f < n plus, ( ) 1 0 f > , exist un unic ( ) ,1
n
x pentru care
( ) 0
n
f x = ; evident ( )
n
x este astfel mrginit. Dac
1 n n
x u v x
+
= = ,
deducem 0 ln
n n n n
v
u v u v v u
u
= > , contradicie. Aadar
1
,
n n
x x n

+
< N adic irul este i strict monoton. Acum teorema lui
Weierstrass i apoi trecere la limit n relaia ( ) ( ) ln 0
n
n n
x x + = , de unde
[ ] ln 0, 0,1 1.
n
L L L L + = =
XII.111 Pentru fiecare t Rse consider
3 2
: , ( )
t t
f f x x t x = + R R .
a) S se arate c
t
f este bijectiv;
b) S se arate c funcia
1
: , ( ) (1)
t
g g t f

= R R este continu n 0.
* * *
Surs: Varianta 93, subiecte bacalaureat 2008.
52
XII.112 Se consider o funcie [ ] : 1,1 f Rde dou ori derivabil cu
(0) 0 f = i '(0) 1. f =
a) S se arate c dac [ ] '( ) 1, 0,1 f x x , atunci
1 1
( ) , ,1
2 2
f x x




;
b) S se calculeze ( )
1
0
lim 1 ( ) x
x
f x

+ .
* * *
Soluie: a) Considerm funcia derivabil [ ] : 0,1 , g R ( ) ( ) g x f x x = .
Avem evident (0) 0 g = i [ ] '( ) 0, 0,1 g x x , aadar g este cresctoare.
Presupunem c exist
0
1
,1
2
x



pentru care ( )
0
1
2
f x < ; deducem c
( )
0 0
1
0
2
g x x < , contradicie. b) limita cerut este egal cu e.
XII.113 S se determine cel mai mare numr real a pentru care
2
1 ln , . x a x x + + R
* * *
Soluie: Se consider funcia : f R R,
2
( ) 1 ln f x x a x = + .
Deoarece
1
2
x = este punct de minim, se impune
1
0
2
f




, de unde
ajungem la
3 ln 2
.
2
a
+
=
XII.114 S se determine numrul soluiilor reale ale ecuaiei
x
x e m

+ = , unde m este un numr real oarecare.


* * *
Soluie: Se consider : f R R, ( )
x
f x x e

= + , se studiaz variaia
acesteia i se ajunge la : pentru 1 m< , ecuaia nu are soluii; pentru 1 m = ,
ecuaia are o rdcin dubl, iar pentru 1 m > , ecuaia are dou rdcini
reale distincte.

w
w
.
n
e
u
t
r
i
n
o
.
r
o
53
Concursul Judeean al Revistei de Matematic
Cara-Severin, Ediia a IV-a
Regulament
Fiecare elev trebuie s rezolve(subliniem din nou: singur!) ct mai
multe probleme de la clasa sa, de la clasa precedent sau de la orice clas
superioar. Redactai ngrijit fiecare problem pe cte o foaie separat
(enun + autor + soluie + numele vostru +clasa), completai talonul de
concurs de pe ultima copert a revistei i trimitei totul ntr-un plic format
coal ministerial, adresat astfel: Prof.Lucian Dragomir, Grup colar
Industrial Oelu-Rou, str.Republicii 10-12, 325700, Oelu-Rou, Cara-
Severin (n colul din dreapta jos a plicului), cu meniunea probleme
rezolvate, clasa .(n colul din stnga jos, scriei evident clasa n care
suntei). Colul din stnga sus v este rezervat(expeditor), acolo v scriei
numele, prenumele, adresa. Insistm asupra trimiterilor n plic(nu n folii
de plastic) i asupra respectrii cu strictee a termenelor finale indicate de
fiecare dat-plicurile primite dup data limit nu vor fi luate n
considerare.
La ediia a IV-a a concursului vor fi selectai concurenii n funcie
de punctajele obinute din rezolvarea problemelor publicate n numerele
23, 24, 25 i 26 ale revistei noastre. n jurul datei de 31 ianuarie 2009 se
va ntocmi clasamentul general (prin nsumarea punctelor obinute) i
astfel primii clasai vor fi invitai, mpreun, ca i n acest an, s participe
la concurs; acesta va avea loc n 28 februarie 2009, probabil din nou la
Oelu-Rou.
Subiectele vor fi alese tot din probleme de genul sau chiar din
RMCS, G.M., R.M.T sau ceva ct de ct nou. Vei remarca, desigur, c
unele probleme pe care vi le propunem sunt din numere mai vechi ale
Gazetei Matematice, n sperana c v vom trezi interesul pentru una
dintre cele mai serioase i vechi reviste de matematic din lume.
Abonai-v la Gazeta Matematic, sigur vei avea numai de ctigat!
Din nou, spor la treab tuturor: elevi , profesori, prini sau prieteni!
(Informaii suplimentare se pot obine la: prof. Ovidiu Bdescu, tel.:
0255/225544 sau prof. Lucian Dragomir, tel: 0255/530303 sau
0722/883537, e-mail: lucidrag@yahoo.com).

54
Probleme propuse
(se primesc soluii pn n data de 21 noiembrie 2008)

Clasa a IV-a

IV.115 Diferena a dou numere naturale este cel mai mic numr de trei
cifre pare distincte. Dac din suma lor scdem cel mai mare numr impar
de trei cifre identice obinem succesorul numrului 750. Aflai
desczutul.
Inst.Mariana Mitric, Reia
IV.116 Participnd la Crosul Europei, Andrei a primit la linia de sosire
cartonaul 10, iar Bogdan s-a aflat pe locul 15, numrnd de la sfrit.
tiind c ntre Andrei i Bogdan mai erau doi copii, iar Bogdan a obinut
un timp mai bun dect Andrei, aflai ci sportivi au participat la cros.
Inst.Mariana Mitric,Reia
IV.117 Bunica lui Anton vinde la pia mere i prune. Pentru 6 kg de
mere i un kg de prune, bunica cere 20 de lei, iar pentru 6 kg de prune i
un kg de mere, cere 15 lei. Marcu are 10 lei i nu i plac merele. Cte kg
de prune poate cumpra Marcu de la bunica lui Anton?
Prof. Simina Moica, Arad
IV.118 Bunicul lui Anton are 50 de pomi fructiferi (meri i pruni), iar
bunicul lui Marcu are n livada sa cirei i viini. Numrul merilor este
dublul numrului viinilor, iar numrul prunilor este dublul numrului
cireilor. Dac bunicul lui Marcu ar mai avea 5 cirei, atunci numrul
acestora ar fi egal cu cel al merilor. Putei gsi ci pomi, din fiecare soi,
are fiecare dintre cei doi bunici?
Prof. Simina Moica, Arad
IV.119 a) Care este numrul minim de numere diferite de o cifr care prin
adunare dau un numr mai mare dect 40?
b) n cte feluri poate fi scris numrul 40 ca diferen a dou
numere de cte dou cifre?
* * *
IV.120 Dac a i b sunt numere naturale diferite, se consider c
perechile ( ) , a b i ( ) , b a sunt diferite. S se determine numrul perechilor
( ) , x y de numere naturale pentru care 1 3 7 25 x y + + + + = .
* * *


w
w
.
n
e
u
t
r
i
n
o
.
r
o
55
Clasa a V-a

V.115 Se consider numerele
2009 2 3 2009
1 1 1 1 1 1
1 1 ...
2 3
3 3 3 3
B

= + + + + + +


i
2011 2 3 2011
1 1 1 1 1 1
1 1 ...
5 6
6 6 6 6
C

= + + + + +


. S se determine n

N
pentru care numrul 125 :
n
A B C = se divide cu
2008
10 .
Prof. Delia Marinca, Timioara
V.116 S se determine pentru ce valori ale lui nN numrul
2 n
n se
scrie ca un numr cu 2 n cifre?
Prof. Loreta Ciulu, Reia
V.117 S se determine cifrele x,a,b,c cu a,b,c consecutive, tiind c
cba ax xa = (numerele fiind scrise n baza zece).

Prof. Loreta Ciulu, Reia
V.118 Artai c numrul:
1985 1985 1985 1985 1985
2000 2001 2002 2003 2004 a = + + + + este divizibil cu 10.
Prof. Snefta Vladu, Moldova Nou
V.119 Alina i Roxana i aleg cte un numr.Dac nmulim cu 12
numrul Alinei, obinem numrul Roxanei nmulit cu 10. Dublul
numrului ales de Alina este cu 8 mai mic dect triplul numrului ales de
Roxana.Putei gsi care dintre cele dou prietene i-a ales un numr mai
mare ?
Prof. Simina Moica, Arad
V.120 Pentru orice numere naturale nenule n i m se noteaz
2
( , ) . x n m n n m = +
a) S se arate c pentru orice m

N , exist n

N astfel nct ( , ) x n m
s fie ptrat perfect;
b) S se arate c exist , , 7, 7 m n m n

N astfel nct ( , 7) x n i
(7, ) x m s fie ptrate perfecte;
c) S se gseasc cel mai mic numr natural k pentru care
(7,1) (7, 2) ... (7, ) 2008. x x x k + + + >
Prof. Ovidiu Bdescu, Reia


56
Clasa a VI-a

VI.115 Se consider a i b numere naturale nenule astfel nct 11divide
numrul 5 a b + .S se arate c 11 divide numerele 5 14 a b + i 3 4 a b + .
Prof.Nistor Budescu, Dalboe
VI.116 S se determine cifrele x,a,b,c cu a,b,c numere consecutive, tiind
c cba ax xa = (numerele fiind scrise n baza zece).
Prof. Loreta Ciulu, Reia
VI.117 Determinai cele mai mici numere naturale nenule x,y,z,t i
n, tiind c: 8 10 15 24 ( ) x y z t x y z t n = = = = + + +
Prof. Loreta Ciulu, Reia
VI.118 Un elev a decupat un triunghi oarecare AOB, despre care tia c
( )
0
47 m AOB = .Folosind numai acest ablon, un creion i rigla negradat
(fr raportor), el a reuit s construiasc un unghi cu msura de
o
8 .
Construii i voi acest unghi n aceleai condiii, explicnd cum ai
procedat.
Prof. Delia Marinca, Timioara
VI.119 La o aniversare, gazda a invitat la masa rotund din sufragerie pe
cei 7 copii invitai (fiecare are cel puin 7 ani, te poi nelege cu ei) i care
au suma vrstelor egal cu 60 de ani.
a) S se arate c printre copiii invitai exist cel puin unul care are cel
puin 9 ani;
b) S se arate c exist 2 vecini la mas care au suma vrstelor cel
puin egal cu 17 ;
c) Este posibil ca exact 3 dintre copii s aib fiecare cte 10 ani?
d) Este posibil ca exact 4 dintre copii s aib fiecare cte 10 ani?
Augustin i Ioan Septimiu Dinulic, elevi, Caransebe

VI.120 Pe o dreapt d se consider, n aceast ordine, punctele A,B,C,D
astfel nct B este mijlocul lui ( ) AC , C este mijlocul lui ( ), BD iar
4 . BC cm = Dac E este mijlocul lui ( ) BC , iar F i G sunt separate de
dreapta d astfel nct , , 4 , 6 FE d GD d FE cm GD cm = = , s se
compare lungimile segmentelor ( ) AF i ( ). EG
Prof. Delia i Adrian Dragomir, Caransebe




w
w
.
n
e
u
t
r
i
n
o
.
r
o
57
Clasa a VII-a

VII.115 S se determine numerele naturale nenule a,b,c pentru care

2 3 4
2 3 4
a b c
a b c
= =
+ + +
.
Prof.Nistor Budescu, Dalboe
VII.116 S se determine numerele naturale x,y,z pentru care
2 2 2 2336.
x y z
+ + =
Olimpiad Vietnam
VII.117 Se d ABC isoscel (AB=AC) cu ( ) m BAC =

( )
0 0
90 180 < < n care [ ] AD este nlime, iar (AE i (AF
bisectoarele BAD , respectiv CAD , ( ) ( ) DC F BD E , .
Fie ( ) AD M i ( ) ( ) AD MD .
a) Artai c : AE MF ||
b) Determinai astfel nct AC MF
c) Pentru determinat la punctul b) precizai natura AMC .
Prof. Loreta Ciulu, Reia
VII.118 a) S se arate c nu exist numere ntregi a i b pentru care

2 1 2 3
1 1
a b
b a
+ +
=
+ +
;
b) S se determine numerele ntregi a i b pentru care

2 1 1
.
1
a a
b b
+ +
=
+

Prof. Simina Moica, Arad
VII.119 S se determine numerele naturale a,b,c pentru care
2 . ab bc ca abc + + = +
* * *
VII.120 n triunghiul ABC se consider bisectoarea ( AE a unghiului
BAC X , ( ) E BC i punctele ( ), D AB F AE CD astfel nct
2 , 3 DB AB EC BC = = i 4 . FC AB =
S se determine msurile unghiurilor triunghiului ABC.
Prof.Lucian Dragomir, Shortlist ONM, 2008




58
Clasa a VIII-a

VIII.115 S se arate c : 17 15 2 13 2 14 12 10 + > .
Prof. Lucian Dragomir, Oelu-Rou

VIII.116 Se consider trapezul isoscel ABCD n care // , AB CD
2 AB CD = . Se noteaz cu M i N mijloacele bazei mari, respectiv bazei
mici, iar P un punct pe dreapta MN pentru care

BPM ABC . S se arate
c dreapta AP este perpendicular sau pe DM sau pe CM.
Prof.Nistor Budescu,Dalboe

VIII.117 Fie triunghiului ABC oarecare i M un punct arbitrar n planul
su. Notm cu
3 2 1
, , G G G centrele de greutate ale triunghiurilor MBA,
MBC respectiv MCA. S se arate c triunghiul
3 2 1
G G G are arie constant
oricare ar fi poziia punctului M n plan, [ ] [ ] [ ] M AB BC CA .
Prof. Loreta Ciulu, Reia

VIII.118 Artai c oricare ar fi numerele naturale m, n, p, q, numrul:
2 35 4 52 5 69 7 86 1
m n p q
N = + + + este divizibil cu 17.
Prof. Delia Marinca, Timioara
VIII.119 Ptratele ABCD i BEFC au latura comun BC de lungime 2.Se
noteaz [ ] [ ] { } AF BC O = i [ ] [ ] { } AF BD G = .
a) S se arate n cel puin trei moduri c exist a astfel nct
[ ]
2
DBF a = A ;
b) S se arate n cel puin dou moduri c O este mijlocul lui [ ] AF ;
c) S se calculeze [ ] ABF A i [ ]. ABG A
Prof. Simina Moica, Arad
VIII.120 Exist numere naturale n pentru care 2 3 10
n n n
+ + este ptrat
perfect ?
Prof.Cristinel Mortici,Trgovite






w
w
.
n
e
u
t
r
i
n
o
.
r
o
59
Clasa a IX-a
IX.115 S se arate c dac ( ) , 0, a b ,atunci

2 2
4 1 1 1
a b ab
a b ab
+ +
+
.
Prof.Nistor Budescu,Dalboe
IX.116 S se rezolve ecuaia :
1 5 1
3 6 2
x x x + +
=


,unde
[ ] a reprezint partea ntreag a numrului real a.
Prof.Nistor Budescu,Dalboe

IX.117 S se determine numerele naturale nenule a,b,c pentru care

1 1 1
1 1 1 2
a b c

+ + + =


.
Olimpiad Marea Britanie
IX.118 Considerm expresia: ( )
1 ...
1 ...
2 99 100
2 99 100
+ + + + +
+ + +
=
x x x x
x x x x
x E
i numrul real
( ) ( ) ( )
( )( )
0, 2 0, 3 0, 4
1 2 1 2

+ +
=
+
. Calculai ( ) E .
Prof. Loreta Ciulu, Reia
IX.119 S se dea un exemplu de ecuaie cu coeficieni ntregi care are o
rdcin egal cu 1 2 3. + +
Prof. Delia i Adrian Dragomir, Caransebe
IX.120 Se consider o funcie : f N N N care satisface proprietile :
a) ( ) , ( , ) ( , ) 1, , ; f x f x y f x y x x y = + + N
b) (0, ) 1, ; f y y y = + N
c) ( 1, 0) ( ,1), . f x f x x + = N
S se calculeze (1, 4). f
Prof. Alfred Ekstein, Viorel Tudoran, Arad

Clasa a X-a
X.115 S se arate c exist un punct P n interiorul unui triunghi ABC
astfel nct

( ) ( ) ( ) m ABP m BCP m CAP = = dac i numai dac triunghiul
ABC este echilateral.
Prof.Nistor Budescu,Dalboe
60
X.116 S se determine perechile ( ) , a b de numere ntregi pentru care

2
log 2 a b + = .
Prof. Simina Moica, Arad
X.117 S se calculeze partea ntreag a numrului

6 11 36 216
log 11 log 36 log 216 log 1296. a = + + +
Prof. Alfred Ekstein, Viorel Tudoran, Arad
X.118 S se determine 0 x > pentru care
81
log
27 6 0.
x
x x + =
Prof. Alfred Ekstein, Viorel Tudoran, Arad
X.119 S se exprime
49
log 8 n funcie de
14
log 42 a = i
14
log 36 b = .
Prof. Delia i Adrian Dragomir, Caransebe
X.120 S se determine numerele naturale m i n pentru care egalitatea
2 2 n m n m
x x x x x = este adevrat pentru orice 0, 1. x x >
Prof. Delia i Adrian Dragomir, Caransebe
Clasa a XI-a
XI.115 Se consider matricea
1 0 2
2 1 2
0 0 1
A


=



.S se calculeze
50
. A
Prof. Simina Moica, Arad

XI.116 Folosind notaiile uzuale pentru un triunghi ABC, s se calculeze
determinantul
2
2
2
1 cos 2
1 cos 2 .
1 cos 2
a A
D b B
c C
=
Prof. Lucian Dragomir, Oelu-Rou
XI.117 S se arate c numrul 91! a = se divide prin
40
3 . b =
Prof. Alfred Ekstein, Viorel Tudoran, Arad
XI.118 O dreapt variabil care trece prin punctul (2,3) P intersecteaz
axele Ox i Oy n punctele A, respectiv B. S se calculeze minimul ariei
triunghiului AOB.
Prof. Alfred Ekstein, Viorel Tudoran, Arad
XI.119 S se determine limita irului ( )
1
n
n
a

definit prin
1
1 a = i

2
1
5 4 , .
n n
a a n

+
= + N
Prof. Alfred Ekstein, Viorel Tudoran, Arad
w
w
.
n
e
u
t
r
i
n
o
.
r
o
61
XI.120 Se consider irul ( )
n
n
x
N
care satisface condiiile:
a) ( ) 0, , ;
n
x n N
b)
1
sin cos 1, .
n n
x x n
+
+ = N
S se arate c irul este convergent i s se determine limita sa.
Olimpiad Cluj, 1983
Clasa a XII-a
XII.115 S se determine numerele prime p pentru care exist numerele
ntregi x,y astfel nct :
2
1 2 p x + = i
2 2
1 2 p y + = .
Olimpiad Germania
XII.116 S se arate c pentru orice ] 1 , 0 [ , , c b a are loc inegalitatea :
(1 )(1 )(1 ) 1
1 1 1
a b c
a b c
b c c a a b
+ + +
+ + + + + +
.
Prof.Ecaterina Zsibriczki,Boca
XII.117 Se definete pe Ro lege de compoziie " " care satisface
condiiile : 1)
1
1 , ;
3 2
a a
a
+
=


R
2) ( ) ( ) ( ) * * , , , . a b c a c b c a b c = R
S se calculeze 10*14 .
Prof.Lucian Dragomir, Oelu-Rou
XII.118 Se consider o funcie [ ) ( ] : 0, 0,1 f derivabil i
bijectiv,care admite o primitiv F cu (0) 0 F = .S se studiez
injectivitatea i surjectivitatea funciei ( ) ( ) : 0, 1, , ( )
( )
x
g g x
F x
= .
Prof.Lucian Dragomir,Oelu-Rou
XII.119 S se determine ( )
13 9 8 4
19 10 9
, 0, .
1
x x x x
dx x
x x x
+ +

+ + +


Prof.Lucian Dragomir, Oelu-Rou
XII.120 S se arate c un grup ( ) , G cu proprietatea c exist
, a b G astfel nct
2 3 2
ab b a = i
4
b e = , conine un subgrup izomorf cu
grupul ( )
2
, + Z .
Prof.Lucian Dragomir, Oelu-Rou


62
Tabel nominal cu membrii Filialei Cara-Severin a SSMR
cu cotizaia pe anii 2007 , 2008

Nr. Nume,prenume Instituia Localitate 2007 2008
1. Cleiu Marian c. cu clase I-VIII Anina * *
2.
Demetrovici Ion
Romeo
Grup colar Anina * _
3.
Neagoe Petrior Grup colar Anina * *
4.
Pruteanu Silvia Grup colar Anina * *
5.
Almjan Ctlin c. cu clase I-VIII Ramna * *
6.
Boriuc Manuela c. cu clase I-VIII Ocna de Fier * *
7.
Ruva Gheorghe Grup colar Anina _ *
8.
Borchescu Angelica Grup colar Anina _ *
9.
Ciolo Aurelia Grup colar Boca * *
10.
Costa Veronica c. cu clase I-VIII Boca * *
11.
Cerbu Enache c. cu clase I-VIII Tirol * *
12. Cosinschi Carmen coala Special Boca * *
13.
Lupulescu Daniela c. cu clase I-VIII Gherteni * *
14.
Lungu Aurel c. cu clase I-VIII Boca * *
15.
Lungu Emilia c. cu clase I-VIII Bini * *
16.
Tulbure Alexandru c. cu clase I-VIII Fize _ *
17.
Miholcea Dan c. cu clase I-VIII Berzovia * *
18.
Musta Maria Liceul Teoretic
Tata Oancea
Boca * *
19.
Peter Eva Maria coala cu clasele
I-VIII
Boca * *
20.
Lacu Maria c. cu clase I-VIII Verme * *
21.
Mustei Liliana Liceul Teoretic
Tata Oancea
Boca * *
22.
Medoia Gheorghe Grup colar Boca * *
23.
Iatan Rodica Liceul Teoretic
Tata Oancea
Boca * *
24.
Rudneanu Iosif coala Special Boca * *
25.
Stniloiu Nicolae Grup colar Boca * *
26.
Stniloiu Manuela c. cu clase I-VIII Boca * *
27.
Todor Veronica Liceul Teoretic
Tata Oancea
Boca * *
28.
Todor Ioan Liceul Teoretic
Tata Oancea
Boca * *
29.
Zibriczki Ecaterina Grup colar Boca * *
30.
Staicu Lenua c. cu clase I-VIII odea _ *
31.
Seracin Ioan c. cu clase I-VIII Mureni _ *
32.
Tina Ana c. cu clase I-VIII Mureni _ *
33.
Surulescu Ion c. cu clase I-VIII Lpunicel * *
w
w
.
n
e
u
t
r
i
n
o
.
r
o
63
34.
Horescu Ioan c. cu clase I-VIII Tople * *
35.
Roman Simion c. cu clase I-VIII Tople * *
36.
Vela Anioara c. cu clase I-VIII Domanea * *
37.
Sadovan Nistor coala cu clasele
I-VIII
Cornea * *
38.
Haracicu Maria c. cu clase I-VIII Bile
Herculane
* *
39.
Opranescu Angela c. cu clase I-VIII Bile
Herculane
* *
40.
Ttucu Anton c. cu clase I-VIII Iablania * *
41.
Rostescu Constantin c. cu clase I-VIII Globu-
Craiovei
* *
42.
Ptracu Zaharia c. cu clase I-VIII Cuptoare * *
43.
Iliescu Sabin c. cu clase I-VIII Mehadica * *
44.
Popescu Adrian Lic.Tehnologic Mehadia * *
45.
Lalescu Tiberiu Lic. Tehnologic Mehadia * *
46.
Ienea Maria Lic.Tehnologic Mehadia * *
47.
Vasile Mihaela Lic.Tehnologic Mehadia * *
48.
Golopena Marius Liceul Hercules Bile
Herculane
* *
49.
Bolbotin Constantin Liceul Hercules Bile
Herculane
* *
50.
Mirulescu Maria Lic.Pedagogic Caransebe * *
51.
Ivacu Nicoleta Lic.Pedagogic Caransebe * *
52.
Humia Dorina Lic.Pedagogic Caransebe * *
53.
Hurduzeu Diana Lic.Pedagogic Caransebe * *
54.
Mandrei Ana Lic.Pedagogic Caransebe * *
55.
Buzescu Antoanela Lic.Pedagogic Caransebe * *
56.
Moatr Lavinia Lic.Pedagogic Caransebe * *
57.
Dragomir Delia Lic. Traian Doda Caransebe * *
58.
Dragomir Adrian Lic. Traian Doda Caransebe * *
59.
Didraga Iacob Lic. Traian Doda Caransebe * _
60.
Bistrian Ana Grup colar Caransebe * *
61.
Didraga Elena Grup colar Auto Caransebe * _
62.
Stvroiu Eugen coala Gen.2 Caransebe * *
63.
Corci Carina coala Gen.2 Caransebe * *
64.
Corci Sebastian coala Gen.2 Caransebe * *
65.
Miu Bocicariu Janet coala Gen.8 Caransebe * *
66.
Dragomir Gorici c. cu clase I-VIII Armeni * *
67.
Ciuc Sorin c. cu clase I-VIII Rusca
Teregova
* *
68.
Liuba Ioan c. cu clase I-VIII Vrciorova * *
69. Gherga Petru c. cu clase I-VIII Vlioara * _
70.
Bob Adam c. cu clase I-VIII Poiana _ *
71.
Isac Daniel Grup colar Auto Caransebe _ *
64
72.
Liuba Ovidiu Grup colar Auto Caransebe _ *
73.
Dragot Ana Grup colar
Forestier
Caransebe _ *
74.
Snduu Janeta Grup colar
Forestier
Caransebe * *
75. Vasilua Violeta c. cu clase I-VIII Zgujeni * *
76.
Jurj Nuu Grup colar
Forestier
Caransebe _ *
77.
Dragomir Petru Scoala de Arte i
Meserii
Teregova * *
78.
Moldovan Cornel Sacu _ *
79.
Albeanu Vasile Grup colar Ind. M.- Nou * _
80.
Drac Cornelia c. cu clase I-VIII Pojejena * _
81.
Gdea Vasilica Grup colar Ind. M.- Nou * *
82.
Hergane Adam c. cu clase I-VIII Sichevia * _
83.
Huza Vasile c. cu clase I-VIII Coronini * *
84.
Iancovici Camelia c. cu clase I-VIII Socol * *
85.
Iovanovici Cristina Grup colar Ind. M.- Nou * _
86.
Mateescu Milena c. cu clase I-VIII Liubcova * *
87.
Ocanovici Zoran Grup colar Ind. M.- Nou * *
88.
Panici Nadia c. cu clase I-VIII Belobreca * *
89.
Radosavlevici
Mrioara
coala cu clasele
I-VIII
Moldova-
Nou
* *
90. Rinariu Lucica c. cu clase I-VIII Sf.Elena * _
91.
Rujici Iasna Floriana c. cu clase I-VIII Cmpia * *
92.
Schiha Emilia Dana c. cu clase I-VIII Berzasca * *
93.
Scoran Gheorghe Grup colar Ind. M.- Nou * _
94.
Vladu Dumitru Grup colar Ind. M.- Nou * *
95.
Vladu Snefta coala cu clasele
I-VIII nr.3
M.- Nou * *
96.
Voilovici Aurelia Grup colar Ind. M.- Nou * *
97.
Ziman Lcrimioara Grup colar Ind. M.- Nou * *
98.
Unanu Giurcevca coala Gen.6 Reia * *
99.
Simulescu Susana coala Gen.6 Reia * *
100.
Lucaci Marilena coala Gen.6 Reia * *
101.
Dicu Lenua coala Gen.12 Reia * *
102.
Socol Maria coala Gen.12 Reia * *
103.
Goa Anca coala Gen.12 Reia * *
104.
Coand Camelia coala Gen.8 Reia * *
105.
Curescu Simona coala Gen.8 Reia * *
106.
Rdoi Mirela coala Gen.8 Reia * *
107.
andru Marius coala Gen.2 Reia * *
108.
Drghici Mariana coala Gen.2 Reia * *
109.
Ciulu Loreta coala Gen.2 Reia * *
110.
Stanca Floarea coala Gen.2 Reia _ *
w
w
.
n
e
u
t
r
i
n
o
.
r
o
65
111.
Mara Adriana Lic.Art Reia * *
112.
Moco Monica-Ana Lic.Art Reia * *
113.
Bea Maria Lic.M. Eliade Reia _ *
114.
Ioca Lucia Lic.M. Eliade Reia _ *
115.
Dobrian Heckl Alina coala Gen.1 Reia _ *
116.
Avrmescu Irina coala Gen.9 Reia * *
117.
Belci Ion coala Gen.9 Reia * *
118.
Chi Vasile coala Gen.9 Reia * *
119.
Buzil Claudia coala Gen.7 Reia * *
120.
Buzil Mircea Lic.Traian Vuia Reia _ *
121.
Rou Lia coala Gen.7 Reia * *
122.
Rduca Rodica coala Gen.7 Reia * *
123. Bcescu Mirela Lic.Bilingv Caraova * *
124.
Pegulescu Delia Lic.Bilingv Caraova * *
125.
Sorca Gheorghe Lic.Bilingv Caraova * *
126.
Iucu Mircea pensionar Reia * *
127.
Bdescu Ovidiu Lic.T.Lalescu Reia * *
128. Deaconu Tudor CCD Reia * *
129.
Gherghe Elena Lic.D-Tietz Reia _ *
130.
Vlduceanu Cristina Lic.D-Tietz Reia _ *
131.
Vucescu Adela Lic.D-Tietz Reia _ *
132.
Crlugea Ana Lic.D-Tietz Reia _ *
133.
Pistril Ion Dumitru Lic.Dragalina Oravia * *
134.
Lazarov Mihai Lic.Dragalina Oravia * *
135.
Milotin Mirela Lic.Dragalina Oravia * *
136.
icu Maria Gr. c. Agricol Oravia * *
137.
Lazarov Aurica Gr. c. Agricol Oravia * *
138.
Chiri Mircea Gr. c. Agricol Oravia * *
139.
Vornicu Eleonora Gr. c. Agricol Oravia * *
140.
Mcoi Geta c. cu clase I-VIII Ciclova-
Romn
* *
141.
Drghicescu Tomi c. cu clase I-VIII Forotic * *
142.
Bobic Florin c. cu clase I-VIII Naid * *
143.
Paul Sorin-Daniel c. cu clase I-VIII Berlite * _
144.
Iancu Maria coala R.Ladea Oravia * *
145.
Bdoi Marian coala R.Ladea Oravia * *
146.
Curea Nicolae coala R.Ladea Oravia * *
147.
Prvu Camelia coala R.Ladea Oravia * *
148.
Simion Gheorghe coala R.Ladea Oravia * *
149.
Spaiuc Veronica c. cu clase I-VIII Crbunari * _
150.
Stancu Iovan-Miu c. cu clase I-VIII Grdinari * _
151.
Milo Laura c. cu clase I-VIII Ciudanovia * _
152.
Pop Alexandru c. cu clase I-VIII Vrdia * _
153.
Braia Carmen c. cu clase I-VIII Greoni * *
66
154.
Iacob Ionel c. cu clase I-VIII Rcdia * _
155.
Imbri Nicolae c. cu clase I-VIII Sasca
Montan
_ *
156.
Pascariu Ion c. cu clase I-VIII Pta _ *
157.
Suta Floare c. cu clase I-VIII Pta _ *
158.
Berbentea Dnil c. cu clase I-VIII Bozovici _ *
159. Rncu Pavel c. cu clase I-VIII Dalboe * *
160.
Pascariu George Liceul Teoretic Bozovici * *
161.
Gin Nicolae c. cu clase I-VIII opotu Vechi * *
162.
Fuicu Clin c. cu clase I-VIII opotu Nou * *
163.
Miclu Simona SAM Prigor * *
164. Borchescu Marius c. cu clase I-VIII Ef. Murgu * *
165.
Pungil Petru c. cu clase I-VIII Lpunicul M * *
166.
Gin Iosif c. cu clase I-VIII Bozovici * *
167.
Clenescu Iancu c. cu clase I-VIII Bnia * *
168.
Bololoi Maria c. cu clase I-VIII Bozovici * *
169.
Ferdean Arjentia Grup colar Oelu-Rou * *
170.
Srbu Eftimie Grup colar Oelu-Rou * *
171.
Boldea Felicia coala Gen. 3 OeluRou * *
172.
Suciu Daniela coala Gen. 3 OeluRou * *
173.
Iacobescu Anioara coala Gen. 1 OeluRou * *
174.
Drghici Liliana coala Gen. 1 OeluRou * *
175.
Feil Heidi coala Gen. 1 OeluRou * *
176.
Luca Daniela coala Gen. 1 OeluRou _ *
177.
Cecon Iulia Grup colar Oelu-Rou * *
178.
Iuhasz Cornelia c. cu clase I-VIII Zvoi _ *
179.
Florea Viorel c. cu clase I-VIII Rusca
Montan
* *
180.
Ardelean Ion c. cu clase I-VIII Rusca
Montan
* *
181. Dragomir Adriana Grup colar Oelu-Rou * *
182.
Dragomir Lucian Grup colar Oelu-Rou * *
183.
Macovei Daniela Lic.Baptist Reia * *
184.
Szucs Alecxandru Lic.Baptist Reia * *
Facem sublinierea c dintre toi acetia i dintre elevii judeului
nostru, prin SSMR, sunt abonai la Gazeta Matematic doar 40 de
persoane (avem semnale c sunt i civa elevi care s-au abonat
individual). Oricum, numrul abonamentelor este incredibil de mic!
Ateptm pentru anul 2009 cereri de abonamente (la prof. Lucian
Dragomir sau la responsabilii de zon). Revista noastr e destul de
bun(aa credem noi), dar nu se poate n niciun caz substitui revistei
de tradiie i valoare care a fost i este Gazeta Matematic!

w
w
.
n
e
u
t
r
i
n
o
.
r
o
67
Rubrica rezolvitorilor
Punctaje obinute pentru rezolvarea probemelor din
RMCS nr. 24
(n parantez apar punctajele realizate pentru ediia a IV-a a
Concursului RMCS)
V rugm s transmitei redaciei orice greeal aprut (clas,
profesor, coal, etc.). V mulumim pentru nelegere.

Clasa a IV-a

Liceul Hercules Bile Herculane (Inst.Alexa Gai, nv. Doina Zah, nv.
Felicia Adriana Laitin, nv. Mirela Bolbotin) Barbu Cristian 80(120),
Burcin Andreea 80(120), Lupan Corina 50(130), Moag Alecsandru
80(130), Cpn Alexandra Maria (40), Velcan Anca (50), Stanciu Ana-
Maria (60), Stanciu Ani (60),Vlascu Ctlin 80(130), Nicoar Denisa
80(130), Strinescu Andra 80(130), Vtavu-Pepa Clina 80(130).
coala Bolvania (Inst. Mihaela Goan) Vlean Robert 10, tirban
Simona 10(20),Mihilescu Flavius 10(20), Dragu Maria 10(20), Jura
Damarius Ctlin 10(10).
Liceul Pedagogic Caransebe (nv. Elena Minea) Szabo Ciprian 80(80).

Clasa a V-a

coala Ciclova Romn ( Prof. Geta Mcoi) Crciunel Viluu 23(23),
Munteanu Andreea 38(38), Muia Diana 38(38).
coala General 2 Reia (nv. Florica Boulescu)Neau Monica 80(118),
Ursul Larisa Iasmina (20), Ciobanu Anca 70(120), Azap Denisa 40(40),
Vasilovici Camil 70(70).
coala Romul Ladea Oravia (Inst. Liliana Crciun) Balmez Andrada-
Ioana 100(160), Murgu Teodora 80(80).
coala General 9 Reia (nv.Lidia Adamescu, prof. Irina Avrmescu)
Stefan Andrei (55), Buoi Natalia 80(138), Munteanu Ionu 110 (158),
Mcnea Alexandra (55), Pepa Nicoleta (55), Boldea Cristina (55), Cata
Larisa (55), Gai Nadine (59), Burghin Vali (48), Antonescu Florina
(58).
coala General 1 Oelu-Rou (nv. Luminia Orszari, Prof. Heidi Feil)
Szalma Eric 30(30).
coala General nr.3 Oelu-Rou(nv. Irina Crstea, Prof. Felicia
Boldea) Preda Sebastian Mihai (30), Lazr Rzvan Ionu 123(123).
68
Liceul Traian Doda Caransebe (nv.Violeta Ionescu) Iliescu
Alexandru 80(139).
Grup colar Moldova Nou( nv. Anastasia Cristina Stroia) Stnu
Nicolae Eduard (40), Stoleriu Nicolae Denis 60(90), Anghel Alexandru
(40), Toma Monica (40), Costina Cristian (40).

Clasa a VI-a

Liceul Hercules Bile Herculane (Prof. Constantin Bolbotin,
prof.Marius Golopena) andru Ilie Daniel 160(254), Gherghina Liviu
(70), Dancu Anca Ionela 8(48) , Dimcea Alexandra Ana Maria 60(110),
Coman Petre Daniel (97), Trk Bogdan (60), Mihart Georgiana (80),
Ferescu Liana (80), Vlaicu Dana (57), uar Bianca 88(88).
coala Berzasca (Prof. Dana Emilia Schiha) Vulpescu Iulia 126(236).
coala Bozovici (Prof.Maria Bololoi) Mitocaru Patricia 151(269), Iancu
Mara-Timea(110), Ruva Mihaela 134(234), Pervu Georgiana (110),
Nicola Ion Cristian (60), Hnda Mihai (110).
Liceul Eftimie Murgu Bozovici (Floarea Haramuz-uta) Dancea Nicolae
150(212), Drgil Ioan Marian 143(197), Suvei Anca Marinela 154(211).
coala General Dalboe (Prof.Pavel Rncu) unea Lia (64), Motoroga
Elisa Mirela (65).
coala General nr. 6 Reia (Prof. Susana Simulescu) Ciulu Miruna
Dalila 210(350).
coala General nr. 9 Reia (Prof. Ion Belci, Prof. Irina Avrmescu)
Peptan Andrei Valentin 136(266), Pangica Antonio (125), Bercean
Bogdan Alexandru 134(259).
coala General nr. 1 Oravia (Prof. Mariana Iancu, Prof. Marian
Bdoi) Gheorghian Clin 195(325), Dnil Mdlina 248(406).
coala General 3 Oelu-Rou (Prof. Daniela Suciu, Prof. Felicia
Boldea) Bil Cristina (90), Romnu Nicoleta (90), Barbu Daniel (84),
Manea Florina (122), Dulan Ioni (80), Preda Cristina (77), Vladu Alina
125(125).
coala General 1 Oelu-Rou(Prof. Heidi Feil) Guan Iuliana 176(284),
Bu Laurian-Paul 140(267), tefnescu Andrei 235(385), Ra Laura(70),
Iamandei Daiana 137(197), Rou Ionu (60), Trica Alexandru (80).
Grup colar Oelu-Rou(Prof. Iulia Cecon) Olaru Ionu 100(147), Roi
Karmina 70(70), Pascu Dalida 50(50).
Liceul Pedagogic Caransebe (Prof. Dorina Humia, Prof. Antoanela
Buzescu) Bivolaru Iulia Mlina 160(240), Bzvan Rzvan Alexandru
60(90), Bzvan Oana Ctlina 67(97), Dinulic Petru Augustin 190(320),
Dinulic Septimiu 190(320), Dragomir Roberto 50(80).
w
w
.
n
e
u
t
r
i
n
o
.
r
o
69
coala Rusca Teregova(Prof. Sorin Ciuc) Stepanescu Georgeta (72),
Oprian Cristina (42), Blaj Ioan 37(85), Moac Nicolae 75(145).

Clasa a VII-a

coala Bnia (Prof.Iancu Clenescu) Odobaa Daniel (107).
coala Bozovici (Prof. Iosif Gin , Maria Bololoi) Vrancea Andreea
126(213), tefan Ana (60), Munteanu Mdlina (90), Holbot Viorica
(97), Beloescu Cristina (107), Hotac Adina 160(160).
Liceul Pedagogic C.D.Loga Caransebe(Prof. Dorina Humia) Ban
Ioana (60), Stanciu Georgiana Maria (40).
coala General Dalboe (Prof. Pavel Rncu)Bcil Alexandru 50,
Careba Denisa (56).
Grup colar Moldova Nou(Prof. Vasilica Gdea)Oprea Adelina
60(110), Tarsoly Carla 40(90), Beloia Marinela (50).
coala General 2 Reia (Prof. Mariana Drghici) eudan Adina
120(210), Popa Andreea 40(100), Onofrei Iulia 100(158), Drghici Livia
Liliana 160(290),Aghescu Monica Elena 150(210).
coala General nr. 9 Reia (Prof. Irina Avramescu, Prof.Vasile Chi,
Prof. Ion Belci) Peptan Alexandru 133 (243), Colgea Alexandru (60),
Lazr Silviu Ioan 120(330), Muscai Lorena (60).
Liceul Gen.Dragalina Oravia (Prof. Mihai Lazarov) Sava Isabella (58).
coala Romul Ladea Oravia(Prof. Camelia Prvu, Prof. Minodora
Savu) Marocico Flavius 30(30), Serafin Dennis George 140(140).
coala General 1 Oelu-Rou (Prof. Heidi Feil, Prof. Anioara Popa)
Pop Cristian Ionu 100(180), Radu Ionela 115(202), Tutean Patricia
50(100), Boran Cristian (40), Alexa Alexandra 160(160), Bidilici Rzvan
90(90).
coala General 3 Oelu-Rou(prof.Felicia Boldea)Cruu Robert
90(198), Bil Diana 100(162), Tnas Raul 107(174), Preda Gabriela
Dagmar 120(235), Mihu Marius Cosmin 107(107).
Grup colar Industrial Oelu-Rou (Prof. Iulia Cecon) Vrgatu Alina
40(70), Popescu Ana Maria 40(70), Clu Maria 104(154).
coala Vrciorova (Prof.Ioan Liuba) Turnea Ion (70).
coala Rusca Teregova (Prof. Sorin Ciuc) Humia Ileana (48), Humia
Maria 30(30), Humia Cosmin 30(30).
Liceul Traian Doda Caransebe(Prof. Florin Ciocan) Szabo Ildiko
150(150).
Grup colar Construcii Maini Caransebe (Prof. Sebastian Corci)
Bdescu Patricia Liana 160(160).

70
Clasa a VIII-a

coala Bozovici (Prof. Iosif Gin)Barbe Cezara 116(194), Nicola
Alexandra 116(192), Bcil Cristiana 120(176), Curescu Elena Cristina
116(116) .
Liceul Traian Doda Caransebe (Prof. Adrian Dragomir)Stoicnescu
Gelu 130(210).
Liceul Pedagogic C.D.Loga Caransebe (Prof. Maria Mirulescu)
Timofte Tina 58(118).
coala General Dalboe(Prof. Pavel Rncu) Jarcu lorena Maria (80),
Marin Lidia Mdlina (58), Drgil Ctlin Sebastian (60), Cpn
Dana Maria (57).
Liceul Hercules Bile Herculane(Prof.Marius Golopena)Tabugan Dana
120(120).
Grup colar Moldova Nou(Prof. Vasilica Gdea) Punovici Rebeca
40(40).
coala Rusca Teregova Prof. Sorin Ciuc) Codopan Florinela 67(123),
Davidescu Toma (14), Blaj Marinela 57(57), Humia Maria 96(96).
coala General 2 Reia (Prof. Mariana Drghici) Pascu Andra Diana
70(140).
coala General 1 Oelu-Rou (Prof. Heidi Feil)Kroko Lorena
130(237), Kuhn Anne Marie 130(237).
Grup colar Industrial Oelu-Rou (Prof. Adriana Dragomir)
Dumitresc Cecilia 54(116), Nasta Laura 54(116).
coala Vrciorova (Prof. Ioan Liuba) Mran Marius 60(100).

Clasa a IX-a

coala Bozovici (Prof. Maria Bololoi) Borchescu Anamaria (60),
Borozan Florina (48).
coala Bnia (Prof. Iancu Clenescu) Derlean Pavel 67(142).
Liceul Traian Doda Caransebe(Prof. Delia Dragomir, Prof. Lavinia
Moatr)Mocanu Ioana 42(209), Paan Petru 86(210),Matei Sergiu 47(47).
Liceul Pedagogic C.D.Loga Caransebe (Prof. Dorina Humia, Prof.
Antoanela Buzescu) Semenescu Anca 120(227), Borcean Gheorghe (50),
Marta Marian 90(90).
Grup colar Oelu-Rou (Prof. Heidi Feil, Prof. Lucian Dragomir)
Duma Andrei Florin 53(121), Tutean Claudiu 62(62), Buliga Adrian
Denis 57(57), Bugariu Timeea 62(62), Bugariu Rzvan 72(72).


w
w
.
n
e
u
t
r
i
n
o
.
r
o
71
Clasa a X-a

Liceul Traian Doda Caransebe (Prof. Delia Dragomir, Iacob Didraga)
Bona Petru 48(136), Prunar Victor 145(305), Hurduzeu Iconia 75(132),
Todor Elena 70(70).
Liceul Pedagogic C.D.Loga Caransebe(Prof. Antoanela Buzescu)
Marta Sebastian (88).
Liceul Traian Lalescu Reia (Prof. Ovidiu Bdescu)Simion Larisa (60),
Popovici Georgian (87), Meter Sergiu 52(109).
Grup colar Industrial Oelu-Rou (Prof. Lucian Dragomir)Atinge
Carina 57(99), Cococeanu Oana 86(144), tefnig Sebastian 60(98).
Liceul Gen.Dragalina Oravia(Prof.Mihai Lazarov) Pricop Romina
123(163), Persu Daniel 5 (25).
Grup colar Moldova Nou(Prof. Lcrimioara Ziman)Istudor Deian
45(45).
Clasa a XI-a

Colegiul Naional Moise Nicoar Arad (Prof. Ovidiu Bodrogeanu)
Adina Vlad 67(67)
Liceul Tata Oancea Boca (Prof.Ioan Todor) Stniloiu Ovidiu 90(168).
Liceul Hercules Bile Herculane (Prof.Constantin Bolbotin) Stolojescu
Anca (50)
Liceul Pedagogic C.D.Loga Caransebe (Prof. Lavinia Moatr) Moatr
Alexandra (60), Jdioreanu Doriana (27), MiculescuMatei (47), Colan
Clin (47), Blidariu Florentina (52), Timofte Andrei (47), Megan Ligia
(60), Milcu Roxana (108), Enel Ion (57), Dumitrescu Otilia (47),
Ciobanu Claudiu (47), Humia Gheorghe (42), Stefan Emanuel (47),
Carabin Claudia (47), Neamiu Nicoleta (47), Babeu Nicolae (38), Brsan
Mirela (57), Muntian Adriana (47), Sbil Marius (47), Blidariu
Florentina (55), Voinea Alexandra (40), Hromei Diana (57), Teodorescu
Silviu (38).
Liceul Pedagogic C.D.Loga Caransebe (Prof. Antoanela Buzescu)
Murean Ana-Maria (90), Murean Alexandru Ioan (90).
Liceul Traian Doda Caransebe (Prof. Lavinia Moatr, Prof. Iacob
Didraga) iu Mihai (28), Blulescu Bianca Veronica (40), Galamba Ionel
Marinel (48), Aghescu Alina Mihaela (27), Cristescu-Loga Cerasela (58),
Ionacu Simona-Suzzana (45), Florea Maria Adelina (27), Turnea Ana-
Maria (45), Negrei Mihaela (45), Stolojescu Oana (45), Turnea Adasena
(50),Train Anca (40), Rotaru Nicolae (27), Toth Lavinia (28).
Grup colar Industrial Oelu-Rou(Prof. Lucian Dragomir) Bugariu
Dan 46(121), Lupu Vlad (50), Moisescu Mihaela (30).
72
un plic(de fapt foi ntr-o folie de plastic!) nu are indicat absolut nimic:
nume, clas, coal... am aflat acum c este vorba despre Raluca Damian
30(70). Somn uor.
Clasa a XII-a

Liceul Pedagogic C.D.Loga Caransebe(Prof. Lavinia Moatr)Gurgu
Caius (30), Kremer Emanuela (30), Iliescu Marcel (30), Ciortan Marius
(30).
Grup colar Industrial Oelu-Rou(Prof. Lucian Dragomir) Ungura
Drago 60 (130), Buzuriu Alina (30), Dragomir Lucia (30), Beg Apostol
(30).
Liceul Gen. Dragalina Oravia(Prof.Mihai Lazarov) Nezbeda Harald
54(54).

Rugm insistent s respectai regulile de trimitere a
soluiilor exact cum sunt indicate pe ultima copert!





Repetm din nou invitaia fcut de nenumrate ori, colegilor
notri, elevilor notri, de a trimite materiale compatibile cu profilul
revistei noastre(note, articole, probleme, proiecte); pe ct posibil,
toate redactate electronic, folosind pentru orice mic formul sau
relaie matematic, editorul de ecuaii Math.Type, font Times New
Roman, caractere 11. Revista este a noastr, a voastr, a judeului,
a Filialei Cara-Severin a Societii de tiine Matematice din
Romnia, chiar dac uneori d impresia c este a unui cerc
restrns de colaboratori; dac aa vi se pare, ntrebai-v ce
facei pentru a schimba asta. Dac nu, e destul de bine...

S-ar putea să vă placă și